ORTHOPEDIC MCQS BANK WITH ANSWER PATHOLOGY 02
ORTHOPEDIC MCQS BANK WITH ANSWER PATHOLOGY 02
1. A 16-year-old boy has had thigh pain for the past several months. He denies any history of trauma. Examination reveals a large, deeply fixed, soft-tissue mass in the thigh. Laboratory results show an elevated erythrocyte sedimentation rate (ESR) and leukocytosis. A plain radiograph and MRI scan are shown in Figures 1a and 1b. Biopsy specimens are shown in Figures 1c and 1d. What is the most likely diagnosis?
1- Ewing’s sarcoma
2- Osteomyelitis
3- Osteosarcoma
4- Chondrosarcoma
5- Giant cell tumor of bone
PREFERRED RESPONSE: 1
DISCUSSION: Ewing’s sarcoma typically can occur in the diaphysis of the long bones (50% to 55%). It is often accompanied by a large soft-tissue mass. Abnormal findings are common, including a low-grade fever, an elevated ESR, and leukocytosis. The histology is consistent with a small round blue cell tumor. The unique pathology and other findings exclude osteosarcoma. Giant cell tumor and chondrosarcoma have a different histologic appearance and typically are more metaphyseal in location. Chondrosarcoma typically is found in older age groups, has a different histologic pattern, and rarely occurs in the midshaft of the femur.
REFERENCE: Simon MA, Springfield DS, et al: Ewing’s Sarcoma: Surgery for Bone and Soft Tissue Tumors. Philadelphia, PA, Lippincott Raven, 1998, pp 287-297.
2. A 14-year-old boy who is right handed reports right shoulder pain. Radiographs show a lucent lesion of the proximal humeral epiphysis with a narrow zone of transition. Results of an open biopsy confirm the presence of a chondroblastoma. Based on these findings, the next most appropriate step in management should consist of
1- intralesional curettage and bone grafting.
2- intra-articular resection of the proximal humerus and endoprosthetic replacement.
3- intra-articular resection of the proximal humerus and osteoarticular allograft reconstruction.
4- extra-articular resection of the proximal humerus and allograft arthrodesis of the shoulder.
5- observation and serial radiographs.
PREFERRED RESPONSE: 1
DISCUSSION: The patient has a chondroblastoma of the proximal humerus; therefore, the treatment of choice is curettage and bone grafting. Surgical resection of the proximal humerus
is not indicated in the initial treatment of an intraosseous chondroblastoma.
REFERENCES: Mirra JM: Bone Tumors: Clinical, Radiologic, and Pathologic Correlations. Philadelphia, PA, Lea and Febiger, 1989, pp 589-623.
Springfield DS, Capanna R, Gherlinzoni F, Picci P, Campanacci M: Chondroblastoma: A review of seventy cases. J Bone Joint Surg Am 1985;67:748-755.
3. A 25-year-old man has had an insidious onset of left hip pain over the past 11 months. A radiograph, coronal MRI scan, and histopathologic specimens are seen in Figures 2a through 2d. What is the most likely diagnosis?
1- Metastatic lung cancer
2- Osteomyelitis
3- Osteosarcoma
4- Ewing’s sarcoma
5- Eosinophilic granuloma
PREFERRED RESPONSE: 4
DISCUSSION: Ewing’s sarcoma is the second most common primary sarcoma of bone in children and young adults. It is a malignant round cell tumor with uncertain histogenesis. Sheets of uniform small round blue cells with a high nuclear-to-cytoplasm ratio and the absence of osteoid formation differentiate this histologic diagnosis from the other conditions. Immunohistochemical staining and molecular diagnostic studies are useful to verify the diagnosis.
REFERENCE: Sucato DJ, Rougraff B, McGrath BE, et al: Ewing’s sarcoma of the pelvis: Long-term survival and functional outcome. Clin Orthop 2000;373:193-201.
4. What is the most common anatomic location for chondrosarcoma?
1- Hand
2- Distal femur
3- Proximal humerus
4- Spine
5- Pelvis
PREFERRED RESPONSE: 5
DISCUSSION: The most common anatomic location of chondrosarcoma is the pelvis (30%), followed by the proximal femur (20%). Chondrosarcomas appear in the shoulder girdle in 15% of patients but rarely affect the spine or hands.
REFERENCES: Marcove RC, Mike V, Hutter RV, et al: Chondrosarcoma of the pelvis and upper end of the femur: An analysis of factors influencing survival time in one hundred and thirteen cases. J Bone Joint Surg Am 1972;54:561-572.
Simon MA, Springfield DS, et al: Chondrosarcoma: Surgery for Bone and Soft Tissue Tumors. Philadelphia, PA, Lippincott Raven, 1998, p 276.
5. In the spine, osteoblastomas usually originate in the
1- anterior vertebral body.
2- posterior vertebral body.
3- posterior longitudinal ligament.
4- posterior elements.
5- paraspinal muscles.
PREFERRED RESPONSE: 4
DISCUSSION: Osteoblastomas are benign lesions that represent less than 5% of benign bone tumors. Most lesions are located in the spine, followed by the femur, tibia, and skull. Patients with spinal lesions usually have pain that may be associated with scoliosis. The most common location in the spine is within the posterior elements.
REFERENCE: Frassica FJ, Waltrip RL, Sponseller PD, Ma LD, McCarthy EF Jr: Clinicopathologic features and treatment of osteoid osteoma and osteoblastoma in children and adolescents. Orthop Clin North Am 1996;27:559-574.
6. A 64-year-old man with a history of metastatic lung cancer reports increasing right hip pain over the period of several months. Radiographs are shown in Figures 3a and 3b. Initial management should consist of
1- intramedullary fixation.
2- radiation therapy.
3- embolization.
4- segmental resection and proximal femoral replacement.
5- chemotherapy.
PREFERRED RESPONSE: 1
DISCUSSION: The patient has lung cancer metastatic to the right proximal femur. The lesion is large, has destroyed a portion of the cortex, and involves the peritrochanteric region. All of these findings put the patient at high risk for pathologic fracture. The lesion is amenable to intramedullary fixation in the form of some type of reconstruction nail. Chemotherapy alone will not restore the bone stock. Given the extent of the lesion, radiation therapy, chemotherapy, or embolization will not prevent fracture. A proximal femoral replacement would be indicated in patients with bone destruction extending into the femoral head and neck. Bisphosphonates may diminish the risk of subsequent lesions but are not sufficient to treat this high-risk lesion. Radiation therapy should be given postoperatively to prevent further bone destruction.
REFERENCE: Mirels H: Metastatic disease in long bones: A proposed scoring system for diagnosing impending pathologic fractures. Clin Orthop 1989;249:256-264.
7. A 58-year-old woman has had a painless periscapular mass for the past year. An MRI scan and biopsy specimen are shown in Figures 4a and 4b. What is the most likely diagnosis?
1- Elastofibroma
2- Desmoid tumor
3- Nodular fasciitis
4- Malignant fibrous histiocytoma
5- Atrophic serratus anterior
PREFERRED RESPONSE: 1
DISCUSSION: Elastofibroma is a rare tumor that most commonly occurs in adults who are older than age 55 years. The lesions usually grow between the chest wall and the scapula, and 10% are bilateral. Histologic analysis shows that they are composed of equal amounts of elastin and collagen with occasional fibroblasts.
REFERENCES: Briccoli A, Casadei R, Di Renzo M, Favale L, Bacchini P, Bertoni F: Elastofibroma dorsi. Surg Today 2000;30:147-152.
Enzinger FM, Weiss SW: Soft Tissue Tumors, ed 3. St Louis, MO, Mosby Year Book, 1995, pp 165-201.
8. A characteristic genetic translocation has been noted in which of the following tumors?
1- Ewing’s sarcoma
2- Chondrosarcoma
3- Neurofibrosarcoma
4- Osteosarcoma
5- Epithelioid sarcoma
PREFERRED RESPONSE: 1
DISCUSSION: There have been no characteristic gene translocations or rearrangements noted in osteosarcoma, chondrosarcoma, neurofibrosarcoma, or epithelioid sarcoma. In contrast, Ewing’s sarcoma has been noted to have a consistent genetic translocation t(11:22).
REFERENCES: Brockstein BE, Peabody TD, Simon MA: Soft tissue sarcomas, in Vokes EE Golomb HM (eds): Oncologic Therapies. New York, NY, Springer-Verlag, 1999, pp 925-952.
Simon MA, Springfield DS, et al: Management of Surgical Specimens: Surgery for Bone and Soft Tissue Tumors. Philadelphia, PA, Lippincott Raven, 1998, pp 67-75.
9. A 44-year-old man has right hip pain. Radiographs reveal a radiolucent lesion of the femoral head and neck. An MRI scan shows no extraosseous tumor extension. A histopathologic photomicrograph of the biopsy specimen is shown in Figure 5. The chance of metastatic disease developing from this lesion is approximately what percent?
1- 0
2- 5
3- 25
4- 50
5- 75
PREFERRED RESPONSE: 2
DISCUSSION: The risk of pulmonary metastasis from a benign giant cell tumor is estimated to be 5%.
REFERENCE: Rock MG, Pritchard DJ, Unni KK: Metastases from histologically benign giant-cell tumor of bone. J Bone Joint Surg Am 1984;66:269-274.
10. Figures 6a through 6d show the radiographs and biopsy specimens of an 8-year-old girl with leg pain. Management of the lesion should consist of
1- wide resection and salvage of the physis.
2- resection and hemicondylar osteoarticular allograft.
3- cryosurgery.
4- radiation therapy.
5- curettage and bone grafting.
PREFERRED RESPONSE: 5
DISCUSSION: The biopsy specimens show a chondromyxoid fibroma with varying amounts of cartilage, benign fibrous tissue, giant cells, and loose myxoid areas. Chondromyxoid fibroma is a benign active bone lesion that is best treated with aggressive curettage and bone grafting. Although recurrences are common, more aggressive treatment is not warranted initially.
REFERENCES: Wilson AJ, Kyriakos M, Ackerman LV: Chondromyxoid fibroma: Radiographic appearance in 38 cases and in a review of the literature. Radiology 1991;179:513-518.
Beaty JH (ed): Orthopaedic Knowledge Update 6. Rosemont, IL, American Academy of Orthopaedic Surgeons, 1999, pp 167-189.
11. Which of the following is considered the treatment of choice for a chondroblastoma of the proximal tibial epiphysis without intra-articular extension?
1- Observation
2- Curettage and bone grafting
3- Wide local excision
4- Wide local excision and radiation therapy
5- Radiation therapy only
PREFERRED RESPONSE: 2
DISCUSSION: Curettage and bone grafting typically is the preferred method of treatment for chondroblastoma, with local recurrence rates of approximately 10%. Some clinicians advocate the addition of adjuvants such as phenol. Left alone, these lesions can destroy bone and invade the joint. Large intra-articular lesions may require major joint reconstruction. Wide local excision rarely is required to eradicate the tumor. Radiation therapy rarely is indicated and only for unresectable or multiply recurrent lesions.
REFERENCES: Springfield DS, Capanna R, Gherlinzoni F, Picci P, Campanacci M: Chondroblastoma: A review of seventy cases. J Bone Joint Surg Am 1985;67:748-755.
Simon MA, Springfield DS, et al: Chondroblastoma: Surgery for Bone and Soft Tissue Tumors. Philadelphia, PA, Lippincott Raven, 1998, p 190.
12. An otherwise healthy 78-year-old woman has low back and buttock pain. Rectal examination reveals a large sacral mass. Figures 7a and 7b show a CT scan and a sagittal MRI scan of the lumbosacral spine. A biopsy specimen is shown in Figure 7c. What is the most likely diagnosis?
1- Chondrosarcoma
2- Chordoma
3- Metastatic renal cell carcinoma
4- Metastatic breast cancer
5- Infection
PREFERRED RESPONSE: 2
DISCUSSION: A chordoma is a malignant neoplasm originating from remnants of the notochord. It is usually localized to the midline with 50% at the sacrococcygeal area, 35% at the skull base, and 15% at the mobile portion of the spine. Large vacuolated cells (physaliferous cells) are a characteristic of the tumor.
REFERENCES: Mindell ER: Chordoma. J Bone Joint Surg Am 1981;63:501-505.
Samson IR, Springfield DS, Suit HD, Mankin HJ: Operative treatment of sacrococcygeal chordoma: A review of twenty-one cases. J Bone Joint Surg Am 1993;75:1476-1484.
13. Figures 8a through 8c show the lateral radiograph and T1- and T2-weighted MRI scans of a 14-year-old soccer player who reports aching thigh pain. The next most appropriate step in management should consist of
1- CT of the chest.
2- a bone scan.
3- a repeat radiograph in 6 to 8 weeks.
4- repeat MRI in 6 to 8 weeks.
5- an open biopsy.
PREFERRED RESPONSE: 3
DISCUSSION: Although the MRI findings could be misinterpreted as an aggressive soft-tissue process, the periosteal-based ossification on the radiograph in an athlete most likely suggests myositis ossificans. The radiograph should be repeated to see further maturation of the ossification with a typical “zoning” pattern. The zoning pattern is one of peripheral ossification. This is often best seen on a CT scan.
REFERENCES: King JB: Post-traumatic ectopic calcification in the muscles of athletes: A review. Br J Sports Med 1998;32:287-290.
Wang SY, Lomasney LM, Demos TC, Hopkinson WJ: Radiologic case study: Traumatic myositis ossificans. Orthopedics 1999;22:991-995, 1000.
14. A 51-year-old male truck driver has had progressive left hip pain for more than 2 years, and he reports that the pain has become severe in the past 9 months. He is now unable to work because of the pain. Examination reveals that range of motion of the hip is limited to 95 degrees of flexion, 0 degrees of internal rotation, and 20 degrees of external rotation. The plain radiograph, MRI scan, and intraoperative gross photographs are shown in Figures 9a through 9d. Management should consist of
1- synovectomy.
2- local excision with arthroplasty.
3- radiation synovectomy.
4- radiation therapy.
5- radical resection.
PREFERRED RESPONSE: 1
DISCUSSION: The diagnosis is synovial chondromatosis. While the plain radiograph fails to show any calcifications, the MRI scan shows an intra-articular mass that involves the capsule. Grossly multiple granular cartilage nodules are seen. Management should consist of removing all loose bodies along with the synovial membrane.
REFERENCE: Milgram JM: Synovial osteochondromatosis: A histopathological study of thirty cases. J Bone Joint Surg Am 1977;59:792-801.
15. Using methylmethacrylate to fill a biopsy hole in the diaphysis of a femur theoretically achieves what purpose?
1- Local tumor control by chemical cytotoxic effect
2- Local tumor kill from heat generation
3- Minimizes tumor contamination
4- Decreases rate of wound infection
5- Reinforces the bone to prevent fracture
PREFERRED RESPONSE: 3
DISCUSSION: Placing cement over a bone biopsy site prevents tumor contamination by controlling hematoma. Even though the use of cement may impart some strength, the femur is still at significant risk for fracture. The use of bone cement in this manner has not been cleared by the FDA, but many physicians feel that it is appropriate when the patient’s health status has been given careful consideration, and the physician has the necessary knowledge and training. The other options are not important reasons to use methylmethacrylate in biopsies.
REFERENCES: Simon MA, Springfield DS, et al: Biopsy: Surgery for Bone and Soft Tissue Tumors. Philadelphia, PA, Lippincott Raven, 1998, pp 55-65.
Simon MA: Biopsy of musculoskeletal tumors. J Bone Joint Surg Am 1982;64:1253-1257.
16. Gaucher’s disease is manifested by reticuloendothelial system macrophage
accumulation of
1- cholesterol.
2- hydroxyproline.
3- calcium pyrophosphate.
4- homogentisic acid.
5- glucocerebroside.
PREFERRED RESPONSE: 5
DISCUSSION: Gaucher’s disease is characterized by macrophage accumulation of glucocerebroside that is caused by a deficiency of lysosomal enzyme glucocerebrosidase. It is an autosomal-recessive trait and is most commonly found in Ashkenazi Jews. Orthopaedic surgeons see patients with Gaucher’s disease usually because of osteonecrosis of the hip. Calcium pyrophosphate is associated with pseudogout. Hydroxyproline is a breakdown product of collagen and is found in high levels in patients with Paget’s disease. Homogentisic acid is associated with ochronosis (alkaptonuria).
REFERENCES: Beatty JH: Orthopaedic Knowledge Update 6. Rosemont, IL, American Academy of Orthopaedic Surgeons, 1999, pp 247-252.
Goldblatt J, Sacks S, Beighton P: The orthopedic aspects of Gaucher disease. Clin Orthop 1978;137:208-214.
Beutler E: Gaucher’s disease. N Engl J Med 1991;325:1354-1360.
17. A 14-year-old boy reports pain in the distal thigh. He denies any history of trauma. Examination reveals tenderness and swelling of the distal thigh without effusion. A radiograph and CT scan are shown in Figures 10a and 10b. A biopsy specimen is shown in Figure 10c. Management should consist of
1- wide excision with neoadjuvant chemotherapy.
2- radiation therapy.
3- curettage and bone grafting.
4- observation.
5- chemotherapy followed by radiation therapy.
PREFERRED RESPONSE: 3
DISCUSSION: Based on these findings, the patient has an aneurysmal bone cyst. Frequently, fluid-fluid levels can be detected on MRI or CT images. The histologic results show a lesion that consists of cavernous spaces filled with blood. The lining of the cavity contains spindle cells, multinucleated giant cells, and reactive bone. Curettage and bone grafting is the preferred treatment method. Without treatment, these lesions can become quite large and destructive. Radiation therapy is not recommended for resectable lesions. Chemotherapy is not required for these benign lesions.
REFERENCES: Simon MA, Springfield DS, et al: Common Benign Bone Tumors: Surgery for Bone and Soft Tissue Tumors. Philadelphia, PA, Lippincott Raven, 1998, pp 194-200.
Wold LA, et al: Atlas of Orthopaedic Pathology. Philadelphia, PA, WB Saunders, 1990, p 232.
18. The dose of the chemotherapeutic agent doxorubicin (Adriamycin) is limited by which of the following factors?
1- Cardiotoxicity
2- Nephrotoxicity
3- Hepatotoxicity
4- Bone marrow suppression
5- Peripheral neuropathy
PREFERRED RESPONSE: 1
DISCUSSION: Doxorubicin is a chemotherapeutic agent that is best known for dose-limiting cardiotoxicity. This is related not only to the cumulative dose, but also the rate of infusion.
REFERENCE: Brockstein BE, Peabody RD, Simon MA: Soft tissue sarcomas, in Vokes EE, Golomb HM (eds): Oncologic Therapies. Berlin, Springer, 1999, pp 925-952.
19. A previously asymptomatic 12-year-old girl sustained a direct blow to the right lateral knee from a baseball bat. Examination reveals an area of ecchymosis and tenderness over the lateral thigh. The patient can walk without pain, but range of motion of the knee causes discomfort. Plain radiographs of the knee are shown in Figures 11a and 11b. To address the bone lesion, management should consist of
1- a three-phase bone scan.
2- CT.
3- MRI.
4- a repeat examination in 6 weeks.
5- a biopsy.
PREFERRED RESPONSE: 4
DISCUSSION: The plain radiographs reveal a pedunculated osteochondroma with a fracture. There is a bony growth in the metaphysis of a long bone, on a stalk that is directed away from the nearby epiphysis. On the AP view, the host cortical and medullary bone are shown as “blending” with lesional bone. There is also a fracture through the lesion. Based on these radiographic findings, the diagnosis is an osteochondroma; therefore, initial management of an acute fracture of an osteochondroma is symptomatic treatment alone. Additional imaging studies are not indicated in this patient. At times it may be difficult to distinguish a sessile osteochondroma from a parosteal osteosarcoma. In the latter case, the host medullary bone and lesion bone are not confluent. A CT scan may be helpful to distinguish if the host medullary and cortical bone are confluent with the lesion.
REFERENCE: Davids JR, Glancy GL, Eilert RE: Fracture through the stalk of pedunculated osteochondromas: A report of three cases. Clin Orthop 1991;271:258-264.
20. What is the most common pediatric soft-tissue sarcoma?
1- Ewing’s sarcoma
2- Liposarcoma
3- Rhabdomyosarcoma
4- Malignant fibrous histiocytoma
5- Fibrosarcoma
PREFERRED RESPONSE: 3
DISCUSSION: Soft-tissue sarcomas are the sixth most common cancer in children. Rhabdomyosarcoma is the most common type of pediatric soft-tissue sarcoma. Nearly 50% of rhabdomyosarcomas are diagnosed in children who are age 5 years or younger. Unfortunately, there has not been a significant increase in survival in children with metastatic rhabdomyosarcoma despite aggressive therapy including multiple-drug chemotherapy regimens.
REFERENCE: Wexler LH, Helman LJ: Pediatric soft tissue sarcomas. CA Cancer J Clin 1994;44:211-247.
21. A 53-year-old man has a 4- x 5-cm high-grade soft-tissue sarcoma in the midthigh. As part of the staging evaluation, regional nodes should be assessed by
1- CT of the pelvis and groin.
2- sentinal node biopsy.
3- clinical examination.
4- fine needle aspiration.
5- prophylactic inguinal node dissection.
PREFERRED RESPONSE: 3
DISCUSSION: In general, soft-tissue metastases to regional nodes are a relatively rare occurrence (less than 5% overall). The incidence of lymphatic metastasis is highest for synovial sarcoma, rhabdomyosarcoma, clear cell sarcoma, and epithelioid sarcoma. Regional nodes should be assessed clinically. CT is not used to routinely assess regional nodes. Evaluation of a sentinal node is not indicated because of the low incidence of regional nodal involvement. Fine needle aspiration may be indicated to assess clinically suspicious nodes. Prophylactic inguinal node dissection is contraindicated because it may lead to unnecessary complications such as lymphedema.
REFERENCE: Sim FH, Frassica FJ, Frassica DA: Soft-tissue tumors: Diagnosis, evaluation and management. J Am Acad Orthop Surg 1994;2:202-211.
22. The risk of local recurrence after surgical resection of a soft-tissue sarcoma is most closely related to
1- the size of the tumor.
2- the grade of the tumor.
3- the site of the tumor.
4- the surgical margin.
5- radiation therapy.
PREFERRED RESPONSE: 4
DISCUSSION: A positive margin is most closely related to subsequent local recurrence. The other factors cited, including the size and site of the tumor, may be related to local recurrence; however, they are more commonly prognostic because of the difficulty in obtaining wide surgical margins about large or proximal tumors. Radiation therapy has been noted to decrease the incidence of recurrence but is not felt to be as important as the surgical margin. The grade of the tumor has more influence on the prevalence of metastatic disease than the incidence of local recurrence.
REFERENCES: Lewis JJ, Leung D, Heslin M, Woodruff JM, Brennan MF: Association of local recurrence with subsequent survival in extremity soft tissue sarcoma. J Clin Oncol 1997;15:646-652.
Meterissian SH, Reilly JA Jr, Murphy A, Romsdahl MM, Pollock RE: Soft-tissue sarcomas of the shoulder girdle: Factors influencing local recurrence. Distant metastases and survival. Ann Surg Oncol 1995;2:530-536.
23. In a patient with a soft-tissue sarcoma treated by wide excision and radiation therapy, the risk of subsequent fracture is probably most influenced by
1- location of the sarcoma in the lower extremity.
2- excision of cortical bone with en bloc resection.
3- a radiation dose of 60 Gy (6,000 cGy) versus 50 Gy (5,000 cGy).
4- the addition of adjuvant chemotherapy.
5- excision of the periosteum.
PREFERRED RESPONSE: 2
DISCUSSION: While most pathologic fractures are in the lower extremity in patients treated for soft-tissue sarcomas by wide excision and adjuvant radiation therapy, risk factors for such fractures are bone resection associated with excision of the tumor and soft-tissue sarcomas of the thigh that require periosteal stripping at the time of resection. Such fractures can occur late, often more than 6 months after surgery, are difficult to treat, and often result in nonunion.
REFERENCES: Bell RS, O’Sullivan B, Nguyen C, et al: Fractures following limb-salvage surgery and adjuvant irradation for soft-tissue sarcoma. Clin Orthop 1991;271:265-271.
Lin PP, Boland PJ, Healey JH: Treatment of femoral fractures after irradiation. Clin Orthop 1998;352:168-178.
24. Figures 12a and 12b show the radiographs of a 50-year-old patient who reports acute knee pain after sustaining a twisting injury while playing tennis. Examination is unremarkable. The next most appropriate step in management should consist of
1- observation.
2- a biopsy.
3- CT of the chest.
4- MRI of the femur.
5- wide local excision.
PREFERRED RESPONSE: 1
DISCUSSION: The radiographs show localized diffuse cortical thickening that is characteristic of melorheostosis. The condition may be monostotic or it may involve many bones in one extremity (monomelic) in the distribution of a sclerotome. Bone scans will show increased uptake at the site or sites of skeletal involvement. Long tubular bones are most commonly involved. Melorheostosis is usually asymptomatic and requires no treatment. On rare occasions, there may be associated soft-tissue contractures.
REFERENCES: Dorfman H, Czerniak B: Bone Tumors. St Louis, MO, Mosby Inc, 1998, pp 1105-1107.
Campbell CJ, Papademetriou T, Bonfiglio M: Melorheostosis: A report of the clinical, roentgenographic, and pathological findings in fourteen cases. J Bone Joint Surg Am 1968;50:1281-1304.
Hove E, Sury B: Melorheostosis: Report on 5 cases with follow-up. Acta Orthop Scand 1971;42:315-319.
25. An adult patient has an 8- x 4- x 10-cm soft-tissue mass located within the adductor compartment of the thigh. Staging studies should consist of
1- MRI and a radiograph of the thigh and CT of the chest.
2- MRI of the thigh, a chest radiograph, and a CT of the pelvis.
3- MRI of the thigh, a total body bone scan, a chest radiograph, and a CT of the pelvis.
4- MRI of the thigh, a total body bone scan, a chest radiograph, and a CT of the abdomen and pelvis.
5- a CT of the chest and thigh, a total body bone scan, and a chest radiograph.
PREFERRED RESPONSE: 1
DISCUSSION: The appropriate staging studies should consist of MRI and a radiograph of the primary lesion and CT of the chest. MRI is superior to CT for soft-tissue imaging. CT may be useful for evaluating the cortex of bone for invasion by tumor. Bone scans are not commonly used because soft-tissue sarcomas rarely metastasize to bone. CT of the abdomen and pelvis is not typically ordered except for possible liposarcoma. With liposarcoma, there may be a synchronous or metastatic retroperitoneal liposarcoma.
REFERENCES: Demetri GD, Pollock R, Baker L, et al: NCCN sarcoma practice guidelines: National Comprehensive Cancer Network. Oncology (Huntingt) 1998;12:183-218.
Pollock R, Brennan M, Lawrence W Jr: Society of Surgical Oncology practice guidelines: Soft-tissue sarcoma surgical practice guidelines. Oncology (Huntingt) 1997;11:1327-1332.
26. An otherwise healthy 75-year-old man has a painful mass in the popliteal fossa of his right knee. A lateral radiograph of the knee, a CT scan of the distal femur, and a histopathologic specimen are shown in Figures 13a through 13c. Management should consist of
1- observation.
2- radiation therapy.
3- radiation therapy and chemotherapy.
4- surgical resection.
5- surgical resection and chemotherapy.
PREFERRED RESPONSE: 4
DISCUSSION: The patient has a parosteal osteosarcoma of the distal femur. The findings of mild knee pain, radiographic evidence of a radiodense mass involving the parosseous space or surface of the distal femur, and histologic findings of a spindle cell lesion forming immature osteoid with little to no necrosis most likely suggest a parosteal osteosarcoma. The treatment of choice is surgical resection.
REFERENCES: Okada K, Frassica FJ, Sim FH, Beabout JW, Bond JR, Unni KK: Parosteal osteosarcoma: A clinicopathological study. J Bone Joint Surg Am 1994;76:366-378.
Campanacci M: Bone and Soft Tissue Tumors. New York, NY, Springer-Verlag, 1990, pp 433-454.
27. What type of metastatic tumor most often has a lytic radiographic appearance?
1- Breast
2- Prostate
3- Lung
4- Osteosarcoma
5- Bladder
PREFERRED RESPONSE: 3
DISCUSSION: Lung carcinoma most often has a lytic radiographic appearance. Bladder and prostate carcinoma are usually blastic. Breast carcinoma can be both blastic and lytic. Thyroid carcinoma may be difficult to visualize radiographically and may be seen only on MRI scans. Metastatic osteosarcomas typically produce bone.
REFERENCES: Frassica FJ, Frassica DA, McCarthy EF, Riley LH III: Metastatic bone disease: Evaluation, clinicopathologic features, biopsy, fracture risk, nonsurgical treatment, and supportive management. Instr Course Lect 2000;49:453-459.
Mulvey RB: Peripheral bone metastasis. Am J Roentgenol 1964;91:155.
28. A 6-year-old boy has leg pain. A radiograph, MRI, CT, and bone scans, and a biopsy specimen are shown in Figures 14a through 14e. What is the most likely diagnosis?
1- Ewing’s sarcoma
2- Osteomyelitis
3- Osteosarcoma
4- Eosinophilic granuloma
5- Fibrous dysplasia
PREFERRED RESPONSE: 2
DISCUSSION: From an imaging point of view, all of the diagnoses are possible. Biopsy results and cultures are necessary to make the diagnosis. The biopsy specimen shows inflammatory cells and necrotic bone, consistent with osteomyelitis.
REFERENCES: Fletcher BD, Hanna SL: Pediatric musculoskeletal lesions simulating neoplasms. Magn Reson Imaging Clin N Am 1996;4:721-747.
Hanna SL, Fletcher BD, Kaste SC, Fairclough DL, Parham DM: Increased confidence of diagnosis of Ewing sarcoma using T2-weighted MR images. Magn Reson Imaging 1994;12:559-568.
29. A 13-year-old girl has had right groin pain for the past 3 weeks. She denies any history of trauma. Examination of the hip reveals no palpable soft-tissue mass or lymphadenopathy, and there is full range of motion. A plain radiograph and MRI scan are shown in Figures 15a and 15b. Biopsy specimens are shown in Figures 15c and 15d. What is the most likely diagnosis?
1- Fibrous dysplasia
2- Osteosarcoma
3- Ewing’s sarcoma
4- Periosteal chondroma
5- Aneurysmal bone cyst
PREFERRED RESPONSE: 1
DISCUSSION: Fibrous dysplasia frequently occurs in the proximal femur. Microfractures and remodeling can lead to the classic “Shepherd’s crook deformity.” The lesion has a ground-glass appearance on plain radiographs. The histologic appearance shows proliferating fibroblasts in a loose spindle cell background. Dysplastic metaplastic trabeculae are arranged in an irregular or “Chinese letter” appearance.
REFERENCES: Enneking WF, Gearen PF: Fibrous dysplasia of the femoral neck: Treatment by cortical bone grafting. J Bone Joint Surg Am 1986;68:1415-1422.
Simon MA, Springfield DS, et al: Common Benign Bone Tumors: Surgery for Bone and Soft Tissue Tumors. Philadelphia, PA, Lippincott Raven, 1998, pp 194-200.
30. What process is often found associated with other neoplasms?
1- Osteomyelitis
2- Osteoblastoma
3- Aneurysmal bone cyst
4- Giant cell tumor
5- Eosinophilic granuloma
PREFERRED RESPONSE: 3
DISCUSSION: Aneurysmal bone cyst may be either a “pattern” or a “diagnosis.” Therefore, aneurysmal bone cyst should be viewed as a diagnosis of exclusion. Hemorrhage into a variety of primary bone lesions (eg, giant cell tumor of bone, chondroblastoma, osteoblastoma, fibrous dysplasia, osteosarcoma, or vascular neoplasms) may result in intralesional, membrane-bone blood-filled cysts. Such secondary changes may be confused with aneurysmal bone cyst, resulting in inappropriate therapy because assessment should be focused on identifying the underlying primary process. The entire specimen should be examined histologically in an effort to locate an underlying primary bone tumor.
REFERENCES: Bonakdarpour A, Levy WM, Aegerter E: Primary and secondary aneurysmal bone cyst: A radiological study of 75 cases. Radiology 1978;126:75-83.
Levy WM, Miller AS, Bonakdarpour A, Aegerter E: Aneurysmal bone cyst secondary to other osseous lesions: Report of 57 cases. Am J Clin Pathol 1975;63:1-8.
Martinez V, Sissons HA: Aneurysmal bone cyst: A review of 123 cases including primary lesions and those secondary to other bone pathology. Cancer 1988;61:2291-2304.
31. A 14-year-old girl reports hip pain that is exacerbated by weight bearing. A radiograph and biopsy specimen are shown in Figures 16a and 16b. The best course of management should be
1- a steroid injection.
2- curettage and bone grafting.
3- radiation therapy.
4- observation.
5- internal fixation without a bone graft.
PREFERRED RESPONSE: 2
DISCUSSION: The patient has a unicameral bone cyst. Because the subtrochanteric part of the femur is a high-stress region, the treatment of choice is bone curettage and grafting.
REFERENCES: Azouz EM, Karamitsos C, Reed MH, Baker L, Kozlowski K, Hoeffel JC: Types and complications of femoral neck fractures in children. Pediatr Radiol 1993;23:415-420.
Hecht AC, Gebhardt MC: Diagnosis and treatment of unicameral and aneurysmal bone cysts in children. Curr Opin Pediatr 1998;10:87-94.
32. A 37-year-old man has left shoulder pain and weakness. Coronal T1- and axial
T2-weighted MRI scans are shown in Figures 17a and 17b. The biopsy specimen is shown in Figure 17c. What is the most likely diagnosis?
1- Abscess
2- Ganglion cyst
3- Nerve sheath tumor
4- Lymphoma
5- Metastatic cancer
PREFERRED RESPONSE: 2
DISCUSSION: A ganglion cyst arising from the posterior labrum of the shoulder is a known cause of suprascapular nerve impingement. The MRI characteristics of low-signal intensity on T1 and high-signal intensity on T2 are seen in lesions with a high fluid content. The histology shows a cavity with a thin fibrous lining.
REFERENCES: Ferrick MR, Marzo JM: Suprascapular entrapment neuropathy and ganglion cysts about the shoulder. Orthopedics 1999;22:430-434.
Post M: Diagnosis and treatment of suprascapular nerve entrapment. Clin Orthop 1999;368:92-100.
33. Of the following factors, which is considered the most important prognostic indicator in soft-tissue sarcomas?
1- Age of the patient
2- Prior excisional biopsy
3- Histologic subtype
4- Superficial versus deep to the fascia location
5- Size of the sarcoma
PREFERRED RESPONSE: 5
DISCUSSION: Histologic grade, the presence or absence of metastatic disease, and tumor size are important prognostic factors. Of the available choices, however, the size of the sarcoma is the most important prognostic indicator. A tumor size of greater than 5 cm is a more important prognostic factor than tumor location. Patients with sarcomas that measure 5 cm or less have nearly identical 3-year survival rates regardless of whether the tumor is subcutaneous or deep. Histologic grade (high versus low) is an important factor. However, histologic subtype frequently is not as important a factor as tumor size.
REFERENCE: Peabody TD, Monson D, Montag A, Schell MJ, Finn H, Simon MA: A comparison of the prognoses for deep and subcutaneous sarcomas of the extremities. J Bone Joint Surg Am 1994;76:1167-1173.
34. Radiographs of a 15-year-old girl with knee pain reveal a radiopaque lesion of the distal femoral metaphysis and epiphysis with a small associated soft-tissue mass. A biopsy specimen shows osteoid and pleomorphic cells with multiple mitotic figures. Staging studies show no other sites of disease. Treatment should consist of
1- above-knee amputation.
2- preoperative chemotherapy, wide resection of the distal femur, and reconstruction.
3- preoperative chemotherapy and above-knee amputation.
4- wide resection of the distal femur and subsequent reconstruction, followed by chemotherapy.
5- curettage and placement of methylmethacrylate.
PREFERRED RESPONSE: 2
DISCUSSION: Based on the location and extent of the tumor, most patients are candidates for limb salvage surgery. The patient has an osteosarcoma; therefore, the treatment of choice is preoperative chemotherapy, wide resection, and reconstruction. There is no difference in survival rates between patients who undergo limb salvage surgery and those who undergo amputation. Curettage and placement of methylmethacrylate is used to treat benign lesions of bone and would not be appropriate in this patient.
REFERENCE: Simon MA, Springfield DS, et al: Common Malignant Bone Tumors: Osteosarcoma. Surgery for Bone and Soft Tissue Tumors. Philadelphia, PA, Lippincott Raven, 1998, pp 265-274.
35. What form of fixation is associated with the highest incidence of osseous union when using segmental allograft reconstruction following tumor resection?
1- Plate and screw fixation
2- Fluted intramedullary rods
3- Interlocking intramedullary rods
4- Cemented intramedullary rods
5- Step-cut osteotomies and interlocking intramedullary rods
PREFERRED RESPONSE: 1
DISCUSSION: Plate and screw fixation of allograft is associated with the highest incidence of union but also the highest prevalence of allograft fracture. Conversely, intramedullary fixation is associated with a higher incidence of nonunion but fewer fractures. Step-cut osteotomies are not associated with a higher incidence of union.
REFERENCE: Vander Griend RA: The effect of internal fixation on the healing of large allografts. J Bone Joint Surg Am 1994;76:657-663.
36. A 51-year-old woman has had progressively increasing right knee pain for the past 6 months. She has a history of metastatic renal cell carcinoma to the lung and the skeletal system. Radiographs are seen in Figures 18a and 18b. The next step in management of the right distal femur lesion should consist of
1- angiography.
2- curettage and cementation.
3- resection and modular distal femoral knee replacement.
4- radiation therapy.
5- a technetium Tc 99m bone scan and AP and lateral radiographs of the entire right femur.
PREFERRED RESPONSE: 5
DISCUSSION: In a patient with known metastatic disease, the surgeon must rule out additional lesions throughout the femur prior to surgical management. Lesions located in the diaphysis or in the peritrochanteric region may influence the surgical procedure.
REFERENCES: Frassica FJ, Gitelis S, Sim FH: Metastatic bone disease: General principles, pathophysiology, evaluation, and biopsy. Instr Course Lect 1992;41:293-300.
Sim FH: Metastatic bone disease of the pelvis and femur. Instr Course Lect 1992;41:317-327.
37. Following resection of malignant tumors, complications related to endoprosthetic reconstruction are most common in what anatomic location?
1- Proximal femur
2- Proximal humerus
3- Proximal tibia
4- Distal femur
5- Distal humerus
PREFERRED RESPONSE: 3
DISCUSSION: It is generally accepted that reconstructions of the proximal tibia are associated with the highest incidence of failure, probably because of poor soft-tissue coverage, the need for extensor mechanism reconstruction, and other anatomic issues. It also may be related to the fact that patients with tumors of the proximal tibia, in general, have a better prognosis and better survival rates than patients with tumors located elsewhere in the body. Reconstructions of the proximal humerus may be more durable because they are not involved in weight-bearing activities.
REFERENCE: Horowitz SM, Glasser DB, Lane JM, Healey JH: Prosthetic and extremity survivorship after limb salvage for sarcoma: How long do the reconstructions last? Clin Orthop 1993;293:280-286.
38. Figures 19a and 19b show the AP and lateral radiographs of an 18-year-old man who has had knee pain for 3 months. Figure 19c shows a histopathologic photomicrograph of the biopsy specimen. Which of the following factors is most likely to affect his survival?
1- The decision to proceed with limb-sparing surgery instead of amputation
2- The presence of a mutated p53 gene
3- Pulmonary nodules detected on a CT scan
4- Location of the tumor
5- Tumor necrosis of 99% following preoperative chemotherapy
PREFERRED RESPONSE: 3
DISCUSSION: The presence of metastatic disease at diagnosis is the most significant prognostic factor in a patient with osteosarcoma. Limb-sparing surgery, when compared with amputation, is not noted to be associated with any difference in the rate of survival. The presence of a mutated p53 gene has been noted in a minority of patients with osteosarcoma; however, the prognostic importance on an individual basis is unclear. Tumor location and a relative poor response to chemotherapy would be secondary to the presence of metastatic disease in estimating a patient’s prognosis.
REFERENCES: Weis L: Common malignant bone tumors: Osteosarcoma, in Simon MA, Springfield DS (eds): Surgery for Bone and Soft Tissue Tumors. Philadelphia, PA, Lippincott-Raven, 1998, pp 265-274.
Mirra JM: Bone Tumors: Clinical, Radiologic, and Pathologic Correlation. Philadelphia, PA, Lea and Febiger, 1989, pp 248-438.
39. What anatomic site is considered at highest risk for pathologic fracture?
1- Humeral neck
2- Humeral diaphysis
3- Subtrochanteric femur
4- Femoral diaphysis
5- Pelvis
PREFERRED RESPONSE: 3
DISCUSSION: The subtrochanteric femur has been identified as an anatomic site that is particularly prone to pathologic fracture. An avulsion fracture of the lesser trochanter is a sign of impending femoral fracture. While the other anatomic locations are also frequently involved in metastatic bone disease, pathologic fractures occur less commonly.
REFERENCES: Simon MA, Springfield DS, et al: Surgery for Bone and Soft Tissue Tumors. Philadelphia, PA, Lippincott Raven, 1998, p 683.
Harrington KD, Sim FH, Enis JE, Johnston JO, Diok HM, Gristina AG: Methylmethacrylate as an adjunct in internal fixation of pathological fractures: Experience with three hundred and seventy-five cases. J Bone Joint Surg Am 1976;58:1047-1055.
40. What clinical finding is associated with the least favorable prognosis in an adolescent patient who has been diagnosed with a high-grade osteosarcoma of the distal femur?
1- A solitary peripheral lung metastasis
2- Tumor crossing the physis
3- Extracompartmental extension
4- Skeletal metastasis
5- Tumor necrosis of 99% after chemotherapy
PREFERRED RESPONSE: 4
DISCUSSION: The presence of synchronous bone disease in young patients carries a dismal prognosis, one that is even worse than the presence of resectable pulmonary metastasis. Many osteosarcomas cross the physis; therefore, this has not been shown to be of prognostic importance. Similarly, the presence of the soft-tissue mass has less prognostic significance.
REFERENCE: Mirra JM: Bone Tumors: Clinical, Radiologic, and Pathologic Correlation. Philadelphia, PA, Lea and Febiger, 1989, pp 344-350.
41. A 65-year-old man has a painful mass of the middle finger. A clinical photograph, lateral radiograph, coronal MRI scan, and biopsy specimen are seen in Figures 20a through 20d. What is the most likely diagnosis?
1- Giant cell tumor
2- Chondrosarcoma
3- Osteosarcoma
4- Infection
5- Paget’s disease
PREFERRED RESPONSE: 2
DISCUSSION: Although the degeneration of an isolated benign cartilaginous lesion into a chondrosarcoma is rare, it occurs in roughly 10% of patients with Ollier’s disease. Pain is the most common symptom of chondrosarcoma. The treatment of low-grade chondrosarcoma ranges from intralesional excision to wide amputation. The intent of the surgery is to remove all the disease to decrease the chance of local recurrence.
REFERENCES: Lee FY, Mankin HJ, Fondren G, et al: Chondrosarcoma of bone: An assessment of outcome. J Bone Joint Surg Am 1999;81:326-338.
Kasser JR (ed): Orthopaedic Knowledge Update 5. Rosemont, IL, American Academy of Orthopaedic Surgeons, 1996, pp 181-193.
42. Figures 21a and 21b show the radiograph and CT scan of a 14-year-old patient with thigh pain. The next most appropriate step in management should consist of
1- blood cultures.
2- oral anti-inflammatory drugs.
3- a biopsy.
4- MRI.
5- a SPECT bone scan.
PREFERRED RESPONSE: 2
DISCUSSION: The radiographs show increased density and reactive bone formation. A faint nidus can be seen on the radiograph but is obvious on the CT scan. The initial therapy for an osteoid osteoma should be oral anti-inflammatory drugs. A biopsy or SPECT bone scan is not indicated because the osteoid osteoma is clearly seen on the CT scan. If the patient fails to respond to nonsurgical therapy, CT-guided radiofrequency ablation or surgical excision is indicated depending on the anatomic location.
REFERENCES: Frassica FJ, Waltrip RL, Sponseller PD, Ma LD, McCarthy EF Jr: Clinicopathologic features and treatment of osteoid osteoma and osteoblastoma in children and adolescents. Orthop Clin North Am 1996;27:559-574.
Kneisl JS, Simon MA: Medical management compared with operative treatment for osteoid osteoma. J Bone Joint Surg Am 1992;74:179-185.
Rosenthal DI, Hornicek FJ, Wolfe MW, Jennings LC, Gebhardt MC, Mankin HJ: Percutaneous radiofrequency coagulation of osteoid osteoma compared with operative treatment. J Bone Joint Surg Am 1998;80:815-821.
43. A 14-year-old boy has a midshaft fibular lesion. Biopsy results are consistent with Ewing’s sarcoma. Following induction chemotherapy, local control typically consists of
1- radiation therapy only.
2- curettage and bone grafting.
3- wide excision.
4- below-knee amputation.
5- observation.
PREFERRED RESPONSE: 3
DISCUSSION: Current treatment regimens for Ewing’s sarcoma typically involve induction chemotherapy followed by local control and further chemotherapy. Local control consists of surgery alone, radiation therapy alone, or a combination of the two. In bones that are easily resectable (or expendable) with wide margins, surgery alone is usually recommended. For areas that are unresectable (ie, large, bulky pelvic tumors), radiation therapy alone is sometimes the preferred method of local control. If surgery is chosen and margins are close, radiation therapy can be used as an adjuvant. Amputation rarely is required for an isolated fibular lesion. Observation without adequate local therapy results in local recurrence.
REFERENCES: Nesbit ME Jr, Gehan EA, Burgert EO Jr, et al: Multimodal therapy for the management of primary, nonmetastatic Ewing’s sarcoma of bone: A long-term follow-up of the First Intergroup study. J Clin Oncol 1990;8:1664-1674.
Simon MA, Springfield DS, et al: Ewing’s Sarcoma: Surgery for Bone and Soft Tissue Tumors. Philadelphia, PA, Lippincott Raven, 1998, pp 287-297.
44. An otherwise healthy 65-year-old man reports thigh pain of insidious onset. He states that the pain is increased with weight bearing and also occurs at night. He denies any history of cancer. Radiographs are shown in Figures 22a and 22b. A bone scan shows an isolated lesion. CT scans of the chest and abdominal are negative for any other lesions. Initial management should consist of
1- stabilization with a locked intramedullary rod.
2- stabilization with a compression plate.
3- chemotherapy.
4- radiation therapy.
5- performing a biopsy.
PREFERRED RESPONSE: 5
DISCUSSION: The patient has a solitary lesion that is at high risk for fracture. While metastatic lesions are most common in this age group, a stabilization procedure is contraindicated until results of a biopsy confirm the presence of a metastasis. Because of the lytic lesion and the associated calcification shown on the radiograph, the most likely diagnosis is a chondrosarcoma. Therefore, any type of stabilization procedure with a rod or plate would compromise a wide surgical excision to remove the entire proximal femur. Radiation therapy and chemotherapy are also contraindicated until biopsy results are obtained. Because of these factors, obtaining a frozen section biopsy specimen is considered the next most appropriate step in management. If a metastatic lesion is confirmed on the frozen section, a stabilization procedure could then be performed under the same anesthetic. Therefore, it is important to have a pathologist available at the time of a biopsy.
REFERENCES: Frassica FJ, Frassica DA, McCarthy EF, Riley LH III: Metastatic bone disease: Evaluation, clinicopathologic features, biopsy, fracture risk, nonsurgical treatment, and supportive management. Instr Course Lect 2000;49:453-459.
Mankin HJ, Mankin CJ, Simon MA: The hazards of the biopsy, revisited: Members of the Musculoskeletal Tumor Society. J Bone Joint Surg Am 1996;78:656-663.
45. What is the most appropriate next step in the work-up of a patient with the asymptomatic lesion shown in Figure 23?
1- Open biopsy
2- Needle biopsy
3- Observation
4- CT-guided radiofrequency ablation
5- Antibiotics
PREFERRED RESPONSE: 3
DISCUSSION: The eccentric metaphyseal location, skeletal maturity, narrow zone of transition, and lack of symptoms suggest a benign process and are consistent with a healed nonossifying fibroma. These lesions typically fill in (ossify) with skeletal maturity, eventually remodeling and disappearing. Radiographic monitoring is indicated. Biopsy is not recommended unless the lesion changes radiographically.
REFERENCES: Marks KE, Bauer TW: Fibrous tumors of bone. Orthop Clin North Am 1989;20:377-393.
Bullough PG, Walley J: Fibrous cortical defect and non-ossifying fibroma. Postgrad Med J 1965;41:672-676.
Skrede O: Non-osteogenic fibroma of bone. Acta Orthop Scand 1970;41:362-380.
46. A 63-year-woman has an elbow flexion contracture. History reveals that she underwent three previous surgeries to remove a malignant fibrous histiocytoma of the forearm. An MRI scan reveals a locally recurrent tumor at the site of the previous surgery. Which of the following is considered the most predictive factor for local recurrence?
1- Tumor grade
2- Tumor size
3- Surgical margin
4- Anatomic site of the tumor
5- Histologic subtype of the tumor
PREFERRED RESPONSE: 3
DISCUSSION: The greatest risk factor for local recurrence is an inadequate surgical margin. The tumor grade, histologic subtype, and size are predictive of systemic relapse. Sarcomas that arise in some anatomic sites, such as the forearm or retroperitoneum, may be more difficult to completely resect compared with other sites. The optimum margin is generally considered to be a cuff of normal tissue beyond the tumor.
REFERENCES: Bell RS, O’Sullivan B, Liu FF, et al: The surgical margin in soft-tissue sarcoma. J Bone Joint Surg Am 1989;71:370-375.
Sadoski C, Suit HD, Rosenberg A, Mankin H, Efird J: Preoperative radiation, surgical margins, and local control of extremity sarcomas of soft tissues. J Surg Oncol 1993;52:223-230.
Wilson AN, Davis A, Bell RS, et al: Local control of soft tissue sarcoma of the extremity: The experience of a multidisciplinary sarcoma group with definitive surgery and radiotherapy. Eur J Cancer 1994;30:746-751.
47. A 19-year-old woman has a painful right knee. A radiograph, MRI scan, CT scan, and histopathologic specimen are shown in Figures 24a through 24d. What is the most likely oncologic stage of the lesion?
1- 2
2- 3
3- IIA
4- IIB
5- III
PREFERRED RESPONSE: 5
DISCUSSION: The patient has a high-grade osteosarcoma of the distal femur with a skip lesion, and pulmonary metastasis is seen on the CT scan. This corresponds to a stage III lesion according to the Musculoskeletal Tumor Society System as adopted from Enneking.
REFERENCE: Enneking WF, Spanier SS, Goodman MA: A system for the surgical staging of musculoskeletal sarcoma. Clin Orthop 1980;153:106-120.
48. Calcitonin acts as an antiresorptive agent by
1- decreasing osteoblast number.
2- decreasing osteoclast activity and number.
3- increasing osteoclast activity.
4- increasing osteoblast activity.
5- increasing osteoblast number.
PREFERRED RESPONSE: 2
DISCUSSION: Calcitonin is a hormone that binds to osteoclasts and acts to decrease both osteoclast activity and number. Calcitonin is most effective in reducing vertebral compression fractures in high-turnover osteoporosis. It is also effective in treating regional osteoporosis. Because of its analgesic effect, it is helpful in treating painful acute compression fractures associated with osteoporosis.
REFERENCE: Lane JM, Nydick M: Osteoporosis: Current modes of prevention and treatment. J Am Acad Orthop Surg 1999;7:19-31.
49. An 8-year-old girl has had a painless enlarging mass of insidious onset in the left thigh for the past 3 weeks. Her mother denies any history of trauma, fever, or disease. Examination reveals a nontender, mobile mass in the left medial thigh. Her gait is normal. Figures 25a through 25d show the frog-lateral radiograph, the axial and coronal T1-weighted MRI scans, and the axial T2-weighted MRI scan. Biopsy results reveal a nonrhabdomyosarcoma soft-tissue sarcoma. The most appropriate treatment should consist of
1- radiation therapy and chemotherapy.
2- surgical drainage and antibiotics.
3- surgical excision with chemotherapy and/or radiation therapy.
4- proximal femoral resection, allograft prosthetic reconstruction, and chemotherapy.
5- serial observation with repeat MRI in 3 months.
PREFERRED RESPONSE: 3
DISCUSSION: In childhood, the more common soft-tissue sarcomas are rhabdomyosarcoma, synovial sarcoma, and fibrosarcoma. Rhabdomyosarcoma, treated with radiation therapy and chemotherapy, is a round cell tumor and is inconsistent with this patient’s histologic findings. Synovial sarcoma can be monophasic or biphasic with both spindle and epithelial-like cells and is associated with the characteristic reciprocal chromosomal translocation of t(x:18)(p11;q11) which is not found in fibrosarcoma. Synovial sarcoma also can be associated with cystic loculated areas best seen in a T2-weighted MRI scan. Nonrhabdomyosarcoma childhood soft-tissue sarcomas are treated with surgical excision in conjunction with chemotherapy and/or radiation therapy. The histology reveals no inflammatory cells to suggest an abscess; therefore, antibiotics and drainage are unnecessary. The MRI scans clearly show a mass of soft tissue and no bone involvement; therefore, proximal femoral resection is not appropriate. Serial observation is not appropriate because of the history of enlargement and insidious onset.
REFERENCES: Enzinger FM, Weiss SW: Soft Tissue Tumors, ed 3. St Louis, MO, Mosby Year Book, 1995, p 757.
Gupta TD, Chaudhuri P (eds): Tumors of the Soft Tissues, ed 2. Stamford, CT, Appleton and Lange, 1998, p 605.
50. Histologically, synovial chondromatosis is characterized by
1- exuberant synovitis (Pannus).
2- loose fragments of articular cartilage embedded in the synovium.
3- ossified cartilage nodules embedded in the synovium.
4- the presence of granulomas in the synovium.
5- hemosiderin deposition in the synovium.
PREFERRED RESPONSE: 3
DISCUSSION: Histologically, there is metaplastic cartilage arising from the synovium. These lobules of zonates hyaline cartilage are of variable size, are embedded within edematous synovium, and protrude into the joint. The lobules calcify and ossify, leading to the characteristic radiographic appearance. Inflammatory synovitis is not characteristic of synovial chondromatosis. The fluid is clear and serosanguin, not blood tinged.
REFERENCES: Milgram JM: Synovial osteochondromatosis: A histopathological study of thirty cases. J Bone Joint Surg Am 1977;l59:792-801.
Murphy FP, Dahlin DC, Sullivan CR: Articular synovial chondromatosis. J Bone Joint Surg Am 1962;44:77.
51. Figure 26a shows the radiograph of a 55-year-old woman who has pain in her right leg after falling. Laboratory studies reveal an elevated alkaline phosphatase level. A biopsy specimen from the proximal tibia is shown in Figure 26b. What is the most likely diagnosis?
1- Metastatic breast cancer
2- Fibrous dysplasia
3- Paget’s disease
4- Hyperparathyroidism
5- Rheumatoid arthritis
PREFERRED RESPONSE: 3
DISCUSSION: Paget’s disease of bone is a metabolic disorder of bone remodeling. The normally coupled process of bone resorption and deposition is lost, resulting in excessive localized bone resorption and compensatory increased bone formation. Pagetic bone tends to be more brittle; therefore, it is susceptible to pathologic fractures and subsequent deformities.
REFERENCES: Lander PH, Hadjipavlou AG: A dynamic classification of Paget’s disease. J Bone Joint Surg Br 1986;68:431-438.
Buckwalter JA, Einhorn TA, Simon SR: Orthopaedic Basic Science, ed 2. Rosemont, IL, American Academy of Orthopaedic Surgeons, 2000, pp 320-369.
52. Radiographs of a pediatric patient reveal a suspected osteosarcoma of the distal femur. Additional staging studies should consist of
1- a radiograph and CT of the chest and a bone scan.
2- a radiograph and MRI of the primary tumor.
3- CT of the abdomen and pelvis.
4- CT of the abdomen, chest, and pelvis, radiographs of the chest and primary tumor, MRI of the primary tumor, and a bone scan.
5- CT of the chest, radiographs of the chest and primary tumor, MRI of the primary tumor, and a bone scan.
PREFERRED RESPONSE: 5
DISCUSSION: CT of the abdomen and pelvis is not part of the staging of osteosarcoma. Staging studies should consist of CT of the chest, radiographs of the chest and primary tumor, MRI of the primary tumor, and a bone scan. The MRI should be obtained prior to the biopsy.
REFERENCE: O’Reilly R, Link M, Fletcher B, et al: NCCN pediatric osteosarcoma practice guidelines: The National Comprehensive Cancer Network. Oncology (Huntingt) 1996;10:1799-1806, 1812.
53. Which of the following factors are considered prognostic of survival in patients with soft-tissue sarcomas?
1- Patient age, surgical margin, and neurovascular invasion
2- Tumor grade, patient age, and tumor depth
3- Tumor grade, tumor depth, and surgical margin
4- Tumor size, patient age, and tumor depth
5- Tumor size, tumor grade, and tumor depth
PREFERRED RESPONSE: 5
DISCUSSION: The factors that are independently prognostic of patient survival are tumor size, tumor grade, and tumor depth (ie, subfascial versus superficial). These factors are the basis for the American Joint Committee on Cancer staging criteria. Patient age and neurovascular invasion are not prognostic. Surgical margin is prognostic for local recurrence but not conclusively for patient survival or metastasis. Metastatic disease is also predictive of survival.
REFERENCES: Cheng EY, Thompson RC Jr: New developments in the staging and imaging of soft-tissue sarcomas. Instr Course Lect 2000;49:443-451.
Fleming ID, et al: Manual for Staging of Cancer/American Joint Committee on Cancer, ed 5. Philadelphia, PA, Lippincott Raven, 1997, pp 149-156.
54. Figures 27a through 27c show the AP radiograph, MRI scan, and biopsy specimen of an otherwise healthy man who has a painful wrist. Serum chemistry studies are normal. What is the most likely diagnosis?
1- Aneurysmal bone cyst
2- Giant cell tumor
3- Renal cell carcinoma
4- Osteomyelitis
5- Brown tumor
PREFERRED RESPONSE: 2
DISCUSSION: The osseous sites most frequently involved by giant cell tumor of bone are the distal femur, proximal tibia, and distal radius with approximately 10% of giant cell tumors involving the distal radius. The goals of treatment are to remove the tumor completely and to preserve maximum function of the extremity.
REFERENCE: Vander Griend RA, Funderburk CH: The treatment of giant-cell tumors of the distal part of the radius. J Bone Joint Surg Am 1993;75:899-908.
55. What is the most common primary malignant tumor of bone in childhood?
1- Osteochondroma
2- Ewing’s sarcoma
3- Osteosarcoma
4- Chondrosarcoma
5- Rhabdomyosarcoma
PREFERRED RESPONSE: 3
DISCUSSION: Osteosarcoma is the most common primary malignant tumor of bone in childhood, followed by Ewing’s sarcoma. Rhabdomyosarcoma is a soft-tissue sarcoma of childhood. Chondrosarcoma rarely occurs in childhood. Osteochondroma is a benign tumor of bone.
REFERENCES: Simon MA, Springfield DS, et al: Osteogenic Sarcoma: Surgery for Bone and Soft Tissue Tumors. Philadelphia, PA, Lippincott Raven, 1998, p 266.
Wold LA, et al: Osteogenic Sarcoma: Atlas of Orthopaedic Pathology. Philadelphia, PA, WB Saunders, 1990, pp 14-15.
56. A 44-year-old woman has bilateral knee pain, and history reveals bilateral hip replacements. Radiographs are seen in Figure 28a, and histopathologic specimens from the total hip replacement are shown in Figures 28b and 28c. Laboratory studies reveal anemia. What is the most likely diagnosis?
1- Osteoarthritis
2- Rheumatoid arthritis
3- Pigmented villonodular synovitis
4- Charcot arthropathy
5- Paget’s disease
PREFERRED RESPONSE: 2
DISCUSSION: Rheumatoid arthritis is an inflammatory arthritis that usually involves multiple joints. Radiologic findings of periarticular erosion, osteopenia, and minimal osteophyte formation favor rheumatoid arthritis over osteoarthritis. Pigmented villonodular synovitis and Charcot arthropathy are more often considered monoarticular diseases. There are no radiographic findings of Paget’s disease.
REFERENCE: Dutkowsky J: Miscellaneous non traumatic disorders, in Crenshaw A (ed): Campbell’s Operative Orthopaedics. St Louis, MO, Mosby, 1992, pp 2007-2012.
57. In addition to pain, which of the following factors are considered most predictive of the risk of pathologic fracture?
1- Anatomic location and pattern of bony destruction
2- Anatomic location and age
3- Anatomic location and weight of patient
4- History of cancer and metaphyseal versus cortical destruction
5- Radiation dose and soft-tissue mass
PREFERRED RESPONSE: 1
DISCUSSION: While guidelines for predicting fracture risk are at best imprecise, the scoring system by Mirels (pain, anatomic location, and pattern of bony destruction) has been shown to be most predictive of fracture risk. Functional pain, peritrochanteric location, and lytic bone destruction are the greatest risk factors for pathologic fracture. The factors of patient weight, age, soft-tissue mass, and location within bone are all of lesser importance.
REFERENCES: Frassica FJ, Frassica DA, McCarthy EF, Riley LH III: Metastatic bone disease: Evaluation, clinicopathologic features, biopsy, fracture risk, nonsurgical treatment, and supportive management. Instr Course Lect 2000;49:453-459.
Mirels H: Metastatic disease in long bones: A proposed scoring system for diagnosing impending pathologic fractures. Clin Orthop 1989;249:256-264.
58. Patients with which of the following primary carcinomas have the shortest overall survival rate after a solitary metastasis to bone?
1- Prostate
2- Breast
3- Thyroid
4- Lung
5- Renal cell
PREFERRED RESPONSE: 4
DISCUSSION: The median survival of patients after discovery of bone metastasis from primary lung carcinoma is shorter compared with other primary sites.
REFERENCE: CA, January/February 2000, vol 50, no. 1 (Cancer Statistics).
59. A 63-year-old woman has a femoral neck fracture. A biopsy specimen obtained from the fracture site at the time of her hemiarthroplasty reveals metastatic carcinoma. Seven days after surgery, she becomes confused and lethargic. Which of the following laboratory values is most likely implicated in the patient’s symptoms at this time?
1- Hemoglobin level of 9.0 g/dL (normal value 11-15 g/dL)
2- Sodium level of 132 mEq/L (normal value 135-145 mEq/L)
3- Potassium level of 5.0 mEq/L (normal value 4.0-5.2 mEq/L)
4- Calcium level of 15 mg/dL (normal value 8.5-10.5 mg/dL)
5- Serum uric acid level of 10 mg/dL (normal value 2.7-7.3 mg/dL)
PREFERRED RESPONSE: 4
DISCUSSION: Although many hematologic and electrolyte abnormalities may be present in a patient with advanced metastatic cancer, an elevated serum calcium level is most commonly associated with confusion. Treatment with hydration, diuretics, and bisphosphonates is recommended.
REFERENCES: Clohishy D: Management of skeletal metastasis in clinical orthopaedics, in Craig E (ed): Operative Orthopaedics. Philadelphia, PA, Lippincott Williams & Wilkins, 1999, pp 994-997.
Mundy GR: Hypercalcemia of malignancy revisited. J Clin Invest 1988;82:1-6.
60. Figure 29a shows the clinical photograph of a 26-year-old woman who has had the leg deformity since birth. She reports difficulty with walking and weight bearing and notes increased discomfort and swelling when the leg is dependent. She denies any history of trauma or family history of a similar disorder. Examination reveals a fixed equinovarus deformity of the foot but no evidence of a limb-length discrepancy. No other cutaneous findings or soft-tissue masses are noted. Sagittal and axial T1- and T2-weighted MRI scans are shown in Figures 29b and 29c. What is the most likely diagnosis?
1- Poliomyelitis
2- Neurofibromatosis (von Recklinghausen disease)
3- Lymphangiomatosis
4- Congenital band syndrome
5- Chronic venous stasis disease
PREFERRED RESPONSE: 3
DISCUSSION: Because the MRI scans show marked dilation and proliferation of lymphatic channels that completely involve all the leg muscles and the clinical photograph shows the severe swelling associated with this disease, the most likely diagnosis is lymphangiomatosis. Poliomyelitis affects the anterior horn cells and manifests as muscle atrophy. Neurofibromatosis can have a similar clinical appearance but usually is associated with other systemic and cutaneous findings. Congenital band syndrome results in amputated or shortened extremities. Chronic venous stasis disease usually is not associated with joint contractures, and typically it affects older individuals. Surgical excision is the only known treatment; this patient underwent an above-knee amputation.
REFERENCES: Berquist TH (ed): MRI of the Musculoskeletal System, ed 3. Philadelphia, PA, Lippincott Raven, 1997, p 771.
Enzinger FM, Weiss SW: Soft Tissue Tumors, ed 3. St. Louis, MO, Mosby Year Book, 1995, p 688.
61. Figures 30a and 30b show the MRI scans of a 51-year-old woman who has had an enlarging soft-tissue mass in her right thigh for the past 18 months. Examination reveals no inguinal adenopathy. Results of a biopsy show a high-grade sarcoma. A bone scan is unremarkable, and a CT scan of the chest shows no evidence of pulmonary metastasis. According to the Musculoskeletal Tumor Society (MSTS) staging system, the tumor should be classified as what stage?
1- 1
2- IA
3- IB
4- IIA
5- IIB
PREFERRED RESPONSE: 4
DISCUSSION: According to the MSTS staging system, soft-tissue sarcomas are staged according to the following factors: grade, site, and metastasis. Roman numerals are used to designate malignant tumors, and Arabic numerals are used for benign tumors. Low-grade malignant tumors are staged as I while intermediate- and high-grade tumors are staged as II. Site is defined as intracompartmental (A) or extracompartmental (B). Any metastasis is staged as III. This patient has a high-grade tumor (II), and the MRI scans show that it is confined to a single compartment; therefore, it is staged as IIA. There is no evidence of metastatic disease.
REFERENCES: Enneking WF, Spanier SS, Goodman MA: A system for the surgical staging of musculoskeletal sarcoma. Clin Orthop 1980;153:106-120.
Peabody TD, Monson D, Montag A, Schell MJ, Finn H, Simon MA: A comparison of the prognoses for deep and subcutaneous sarcomas of the extremities. J Bone Joint Surg Am 1994;76:1167-1173.
Pisters PW, Leung DH, Woodruff J, Shi W, Brennan MF: Analysis of prognostic factors in 1,041 patients with localized soft tissue sarcomas of the extremities. J Clin Oncol 1996;14:1679-1689.
Wunder JS, Healey JH, Davis AM, Brennan MF: A comparison of staging systems for localized extremity soft tissue sarcoma. Cancer 2000;88:2721-2730.
62. An otherwise healthy 45-year-old man has a 3-cm subcutaneous mass on his anterior thigh that is presumed to be a lipoma. Removal of the mass is performed in the office of his primary care physician. Pathologic evaluation shows a high-grade malignant fibrous histiocytoma. Staging reveals no evidence of metastatic disease. Management at this time should consist of
1- resection of the biopsy site and tumor bed, including the deep fascia and a portion of underlying muscle, and skin grafting.
2- resection of the anterior compartment of the thigh and rotational flap, followed by radiation therapy.
3- radiation therapy.
4- chemotherapy and radiation therapy.
5- observation.
PREFERRED RESPONSE: 1
DISCUSSION: Resection of a previously excised soft-tissue sarcoma is recommended. Reoperation is recommended after removal of malignant tumors previously believed to be benign, as approximately one half of the patients will have residual tumor in the re-excised specimen. Observation is not indicated in most patients because local recurrence is likely. Radiation therapy alone may result in long-term local control but is felt to be less effective than reoperation. Chemotherapy and radiation therapy alone are not recommended. Wide excision is the most important factor for local disease control. Radiation therapy after surgical re-excision may also decrease the risk of local recurrence.
REFERENCES: Peabody TD, Monson D, Montag A, Schell MJ, Finn H, Simon MA: A comparison of the prognoses for deep and subcutaneous sarcomas of the extremities. J Bone Joint Surg Am 1994;76:1167-1173.
Giuliano AE, Eilber FR: The rationale for planned reoperation after unplanned total excision of soft-tissue sarcomas. J Clin Oncol 1985;3:1344-1348.
Lewis JJ, Leung D, Espat J, Woodruff JM, Brennan MF: Effect of resection in extremity soft tissue sarcoma. Ann Surg 2000;231:655-663.
Noria S, Davis A, Kardel R, et al: Residual disease following unplanned excision of soft-tissue sarcoma of an extremity. J Bone Joint Surg Am 1996;78:650-655.
63. A 13-year-old girl has had a firm mass and pain in her right shoulder for the past several weeks. She denies any history of trauma. A radiograph and MRI scan are shown in Figures 31a and 31b. Biopsy specimens are shown in Figures 31c and 31d. What is the most likely diagnosis?
1- Osteosarcoma
2- Ewing’s sarcoma
3- Osteochondroma
4- Chondrosarcoma
5- Periosteal chondroma
PREFERRED RESPONSE: 1
DISCUSSION: The patient has osteosarcoma. The radiograph suggests an aggressive primary tumor of bone, and the histology shows malignant cells surrounded by osteoid, classic for osteosarcoma. Ewing’s sarcoma histologically consists of small round blue cells. Osteochondroma and periosteal chondroma occur in the shoulder but have a different histologic pattern and a less aggressive radiographic appearance. Chondrosarcomas rarely occur in children.
REFERENCES: Simon MA, Springfield DS, et al: Osteogenic Sarcoma: Surgery for Bone and Soft Tissue Tumors. Philadelphia, PA, Lippincott Raven, 1998, p 266.
Wold LA, et al: Osteogenic Sarcoma: Atlas of Orthopaedic Pathology. Philadelphia, PA, WB Saunders, 1990, pp 14-15.
64. The use of radiation therapy is most effective in metastatic bone disease from which of the following tumors?
1- Renal cell carcinoma
2- Adenocarcinoma of the lung
3- Adenocarcinoma of the colon
4- Carcinoma of the cervix
5- Lymphoma
PREFERRED RESPONSE: 5
DISCUSSION: Both myeloma and lymphoma are more responsive to radiation therapy. The other types of tumors are relatively more resistant to radiation. Doses of 25 to 50 Gy (2,500 to 5,000 cGy) are usually sufficient for myeloma and lymphoma, while carcinomas frequently require a higher dosage. Large cell lymphoma of bone is usually of B-cell origin and is treated with chemotherapy and radiation therapy.
REFERENCES: Simon MA, Springfield DS, et al: Surgery for Bone and Soft Tissue Tumors. Philadelphia, PA, Lippincott Raven, 1998, p 683.
Riley LH III, Frassica DA, Kostuik JP, Frassica FJ: Metastatic disease to the spine: Diagnosis and treatment. Instr Course Lect 2000;49:471-477.
65. Figures 32a and 32b show the radiographs of an active 13-year-old boy who has persistent left thigh pain and a limp despite a trial of protected weight bearing. Management should consist of
1- curettage and bone grafting.
2- systemic chemotherapy.
3- an intralesional steroid injection.
4- en block resection with autograft reconstruction.
5- low-dose radiation therapy.
PREFERRED RESPONSE: 1
DISCUSSION: The plain radiographs show an eccentric metaphyseal lesion involving a long bone in a skeletally immature patient. The lesion is longer than it is wide, with distinctly lobular outer edges that are sclerotic. These findings are characteristic of a nonossifying fibroma. Small asymptomatic lesions may be followed clinically. Larger lesions that occupy greater than two thirds of the width of the shaft and are located in areas of high mechanical stress such as the femur are more prone to fracture than smaller lesions. Pain is often a sign of impending fracture or the presence of a small fracture that may not be apparent on radiographs. The natural history of the lesion is to resolve over a period of years. The procedure that would allow the patient to return to contact sports is curettage and bone grafting. Intralesional steroid injection has been advocated in the treatment of unicameral bone cysts and eosinophilic granuloma but not nonossifying fibromas. En block resection is not indicated for a benign lesion. Low-dose radiation therapy has been used for eosinophilic granuloma but not for nonossifying fibromas.
REFERENCES: Walker RN, Green NE, Spindler KP: Stress fractures in skeletally immature patients. J Pediatr Orthop 1996;16:578-584.
Arata MA, Peterson HA, Dahlin DC: Pathological fractures through non-ossifying fibromas: Review of the Mayo Clinic experience. J Bone Joint Surg Am 1981;63:980-988.
66. The spread of malignant cells to the vertebrae is often through
1- epidural extension.
2- Batson’s vertebral plexus.
3- the lymphatic system.
4- direct tumor extension.
5- arterial circulation including the artery of Adamkiewicz.
PREFERRED RESPONSE: 2
DISCUSSION: In 1940, Batson described a valveless plexus of veins that extend from the dural venous sinuses of the skull to the sacrum. This system permits retrograde blood flow and enables tumor cells to enter vertebral bodies at multiple levels. Increased intra-abdominal pressure will enhance this retrograde blood flow.
REFERENCE: Batson OV: The function of the vertebral veins and their role in the spread of metastases. Ann Surg 1940;112:138-149.
67. A 10-year-boy has had thigh pain for the past several months. He denies any history of trauma. Examination reveals no soft-tissue mass, and mild tenderness. Figures 33a and 33b show the plain radiograph and MRI scan, and the biopsy specimens are shown in Figures 33c and 33d. What is the most likely diagnosis?
1- Osteomyelitis
2- Eosinophilic granuloma
3- Osteosarcoma
4- Ewing’s sarcoma
5- Lymphoma of bone
PREFERRED RESPONSE: 2
DISCUSSION: The diagnosis is eosinophilic granuloma. The plain radiograph and MRI scan show a lesion in the midshaft of the femur. There is no soft-tissue mass. There is reactive bone about the lesion that suggests a less aggressive tumor. The histology reveals eosinophils in an otherwise bland cellular background with no evidence of mitotic figures or malignant cells to suggest sarcoma. The diagnostic elements are the amphophilic (ie, pale purple) histiocytes with cigar-shaped nuclei, some of which have linear longitudinal grooves. There is no histologic evidence of infection. Lymphoma of bone would be an unusual occurrence in this age group, and the histology is not consistent with that diagnosis.
REFERENCE: Simon MA, Springfield DS, et al: Common Benign Bone Tumors: Surgery for Bone and Soft Tissue Tumors. Philadelphia, PA, Lippincott Raven, 1998, pp 194-200.
68. A 35-year-old patient has right hip pain. Figures 34a and 34b show the coronal MRI scan and the biopsy specimen. What is the most likely diagnosis?
1- Degenerative cyst with collapse
2- Osteonecrosis
3- Chondroblastoma
4- Metastatic carcinoma
5- Clear cell chondrosarcoma
PREFERRED RESPONSE: 2
DISCUSSION: Alcohol abuse, steroids, Caisson disease, sickle cell disease, and radiation therapy are the leading causes of nontraumatic osteonecrosis of the femoral head.
REFERENCES: Arlet J: Nontraumatic avascular necrosis of the femoral head: Past, present, and future. Clin Orthop 1992;277:12-21.
Stulberg BN, Bauer TW, Belhobek GH, Levine M, Davis A: A diagnostic algorithm for osteonecrosis of the femoral head. Clin Orthop 1989;249:176-182.
69. A 16-year-old girl has a painful foot mass. A radiograph, MRI scan, and biopsy specimens are shown in Figures 35a through 35d. What is the most likely diagnosis?
1- Synovial sarcoma
2- Pigmented villonodular synovitis (PVNS)
3- Malignant fibrous histiocytoma
4- Synovial chondromatosis
5- Lyme disease
PREFERRED RESPONSE: 2
DISCUSSION: Synovial sarcoma should always be considered in the differential diagnosis of a foot mass; however, the histopathology shows a typical example of PVNS, with hemosiderin, giant cells, and synovium. Synovial chondromatosis would have metaplastic cartilage in the synovium. The radiograph shows subtle erosion of the lateral cortex of the cuboid, and the MRI scan shows a soft-tissue mass.
REFERENCES: Ghert MA, Scully SP, Harrelson JM: Pigmented villonodular synovitis of the foot and ankle: A review of six cases. Foot Ankle Int 1999;20:326-330.
Jones BC, Sundaram M, Kransdorf MJ: Synovial sarcoma: MR imaging findings in 34 patients. Am J Roentgenol 1993;161:827-830.
Sartoris DJ, Resnick D: Magnetic resonance imaging of pediatric foot and ankle disorders. J Foot Surg 1990;29:489-494.
Scully SP, Temple HT, Harrelson JM: Synovial sarcoma of the foot and ankle. Clin Orthop 1999;364:220-226.
70. What is the most common diagnosis in a patient older than age 40 years with a destructive bony lesion?
1- Metastatic carcinoma
2- Osteosarcoma
3- Lymphoma
4- Multiple myeloma
5- Chondrosarcoma
PREFERRED RESPONSE: 1
DISCUSSION: The most common destructive lesions in a patient older than age 40 years are, in decreasing incidence, metastatic carcinoma, multiple myeloma, lymphoma, and chondrosarcoma. Osteosarcoma is found primarily in younger patients.
REFERENCE: CA, January/February 2000, vol 50, no. 1 (Cancer Statistics).
71. A 78-year-old man being seen for routine follow-up studies reports mild to moderate pain in his left arm. The patient has a history of lung cancer and severe heart disease. A radiograph and a bone scan are shown in Figures 36a and 36b. Treatment should consist of
1- intramedullary fixation.
2- arterial embolization.
3- curettage and cementation.
4- radiation therapy.
5- segmental resection and reconstruction using a diaphyseal metal spacer.
PREFERRED RESPONSE: 4
DISCUSSION: The patient has lung cancer metastatic to the left humerus that is mildly to moderately symptomatic. Radiographically, the lesion is small and is not associated with significant cortical destruction. Metastatic lesions in the upper extremity have a lower incidence of pathologic fracture than lesions in the lower extremity.
REFERENCE: Mirels H: Metastatic disease in long bones: A proposed scoring system for diagnosing impending pathologic fractures. Clin Orthop 1989;249:256-264.
72. Which of the following benign bone lesions can develop lung metastases?
1- Chondroblastoma
2- Chondromyxoid fibroma
3- Fibrous dysplasia
4- Aneurysmal bone cyst
5- Desmoplastic fibroma
PREFERRED RESPONSE: 1
DISCUSSION: Although considered benign bone lesions, lung metastases can develop in giant cell tumors and chondroblastomas. These often can be treated with multiple thoracotomies, resulting in long-term survival.
REFERENCES: Roberts PF, Taylor JG: Multifocal benign chondroblastomas: Report of a case. Hum Pathol 1980;11:296-298.
Bloem JL, Mulder JD: Chondroblastoma: A clinical and radiological study of 104 cases. Skeletal Radiol 1985;14:1-9.
73. The radiographic feature seen in Figure 37 that best indicates a slow-growing process is the
1- deformity of the fibula.
2- wide base of the lesion.
3- lack of joint deformity.
4- medullary canal extending into the lesion.
5- lack of periosteal reaction.
PREFERRED RESPONSE: 1
DISCUSSION: For the fibula (or any bone) to bow, a long-standing process needs to be present. Pressure from a rapid process would cause erosion, not allowing the bone to remodel. The other features are helpful confirmatory findings but also may be associated with aggressive processes. In this patient, the fibular deformity is caused by a sessile osteochondroma of the tibia.
REFERENCES: Lodwick GS, Wilson AJ, Farrell C, Virtama P, Dittrich F: Determining growth rates of focal lesions of bone from radiographs. Radiology 1980;134:577-583.
Moser RP Jr, Madewell JE: An approach to primary bone tumors. Radiol Clin North Am 1987;25:1049-1093.
74. A 52-year-old man has a stage IIB malignant fibrous histiocytoma of the distal femur. Local treatment should consist of
1- wide excision.
2- curettage and bone grafting.
3- curettage and placement of polymethylmethacrylate.
4- prophylactic fixation with an intramedullary rod.
5- radiation therapy only.
PREFERRED RESPONSE: 1
DISCUSSION: Local control of malignant fibrous histiocytoma of bone typically consists of wide excision. Curettage and bone grafting is a procedure with an intralesional surgical margin, with an unacceptable rate of local recurrence. Prophylactic fixation is considered for patients with metastatic disease. Radiation therapy alone is not adequate for local control of this tumor. Neoadjuvant chemotherapy is often used primarily for systemic tumor control.
REFERENCE: Bacci G, Picci P, Mercuri M, Bertoni F, Ferrari S: Neoadjuvant chemotherapy for high grade malignant fibrous histiocytoma of bone. Clin Orthop 1998;346:178-189.
75. A patient undergoes excision of a 3- x 3-cm soft-tissue sarcoma. The final histology results show tumor at the surgical margin. The surgical procedure performed is
classified as
1- intralesional.
2- marginal.
3- wide.
4- wide contaminated.
5- radical.
PREFERRED RESPONSE: 1
DISCUSSION: Surgical margins are classified according to the plane of dissection in relation to the tumor. An intralesional procedure is where the plane of dissection passes through the tumor. A marginal resection involves a resection where the plane of dissection is through the reactive zone of the tumor. A wide resection is through completely normal tissue outside of the reactive zone. A radical resection removes all of the muscles and/or bone of the compartment that are involved by the tumor.
REFERENCE: Enneking WF, Spanier SS, Goodman MA: A system for the surgical staging of musculoskeletal sarcoma. Clin Orthop 1980;153:106-120.
76. A 30-year-old woman has pain in her right hand. The radiograph, CT scan, and biopsy specimen are seen in Figures 38a through 38c. What is the most likely diagnosis?
1- Enchondroma
2- Giant cell tumor
3- Metastatic carcinoma
4- Degenerative cyst
5- Eosinophilic granuloma
PREFERRED RESPONSE: 1
DISCUSSION: An enchondroma is the most common primary tumor of the long bones of the hand. The lesion is usually asymptomatic and often is detected when there is a pathologic fracture.
REFERENCES: Shimizu K, Kotoura Y, Nishijima N, Nakamura T: Enchondroma of the distal phalanx of the hand. J Bone Joint Surg Am 1997;79:898-900.
Takigawa K: Chondroma of the bones of the hand: A review of 110 cases. J Bone Joint Surg Am 1971;53:1591-1600.
77. Which of the following agents have been shown to reduce the incidence of skeletal events in patients with multiple myeloma?
1- Vitamin D and calcium
2- Chelating agents
3- Bisphosphonates
4- Estrogens
5- Progesterones
PREFERRED RESPONSE: 3
DISCUSSION: Bisphosphonates are a class of drugs that act to inhibit osteoclast resorption of bone. It has been shown that patients with multiple myeloma who are treated with bisphosphonates have fewer pathologic fractures than patients who are not treated with bisphosphonates. Vitamin D and calcium are considered appropriate for patients who are at risk for the development of osteoporosis, as is estrogen in selected women. Chelating agents and progesterones have no use in the treatment of patients with multiple myeloma or osteoporosis.
REFERENCES: Berenson JR: Bisphosphonates in multiple myeloma. Cancer 1997;15:1661-1667.
Berenson JR, Lichtenstein A, Porter L, et al: Efficacy of pamidronate in reducing skeletal events in patients with advanced multiple myeloma: Myeloma Aredia Study Group. N Engl J Med 1996;334:488-493.
78. A 12-year-old girl has had progressive left knee pain for the past 4 months. She reports that the pain is unrelated to activity, and she has no history of fever or recent infections. Examination reveals full range of motion of the knee but tenderness along the medial joint line. Plain radiographs and MRI scans are shown in Figures 39a through 39d. A biopsy specimen of the lesion is shown in Figure 39e. Treatment should include
1- curettage.
2- systemic antibiotics.
3- observation and protected weight bearing.
4- chemotherapy.
5- radiation therapy.
PREFERRED RESPONSE: 1
DISCUSSION: The lesion is a chondroblastoma. The plain radiographs show a well-defined radiolucent lesion in the distal femoral epiphysis of a skeletally immature patient. The margins are well defined, suggesting a benign growth. The epiphysis is an unusual location for bone tumors, except for chondroblastomas. Of all chondroblastomas, 95% are located within the epiphysis. The MRI scans show a punctate appearance that is commonly seen in cartilage lesions. The biopsy specimen shows a chondroid lesion with polygonal chondrocytes. These findings are consistent with a chondroblastoma. The natural history of chondroblastomas is for continued growth and bone destruction if left untreated. Treatment should consist of curettage, with or without the use of physical or chemical adjuvants, and bone grafting.
REFERENCE: Springfield DS, Capanna R, Gherlinzoni F, Picci P, Campanacci M: Chondroblastoma: A review of seventy cases. J Bone Joint Surg Am 1985;67:748-755.
79. An open biopsy specimen of a radiodense distal clavicle lesion in a 12-year-old girl shows chronic polyclonal inflammatory cells without granuloma formation. Laboratory studies show that bacterial, fungal, and acid-fast bacillus cultures are negative. Subsequently, a similar lesion is noted in the fibula. The next most appropriate step in management should consist of
1- antiviral therapy.
2- IV broad-spectrum antibiotics.
3- partial claviculectomy and fibulectomy.
4- symptomatic relief with anti-inflammatory drugs.
5- bone marrow aspiration and biopsy.
PREFERRED RESPONSE: 4
DISCUSSION: The most likely diagnosis is chronic multifocal osteomyelitis. This is a culture-negative polyostotic disease that is most commonly found in young people. The treatment of choice is anti-inflammatory drugs. The pathology does not suggest eosinophilic granuloma. Antiviral therapy, broad-spectrum antibiotics, and surgical resection are not indicated for this disease.
REFERENCE: Carr AJ, Cole WG, Roberton DM, Chow CW: Chronic multifocal osteomyelitis. J Bone Joint Surg Br 1993;75:582-591.
80. Figure 40 shows the radiograph of a 30-year-old woman who has a painful elbow. Examination reveals a deformed skull, multiple cafe-au-lait spots, and bone deformities. What is the most likely diagnosis?
1- McCune-Albright syndrome
2- Neurofibromatosis, type I (NF-1)
3- Neurofibromatosis, type II (NF-2)
4- Paget’s disease
5- Ollier’s disease
PREFERRED RESPONSE: 1
DISCUSSION: Findings in patients with McCune-Albright syndrome include polyostotic fibrous dysplasia, multiple cafe-au-lait spots, and precocious puberty. The bone changes in NF-1 resemble nonossifying fibromas, not fibrous dysplasia. NF-2 has little bony change with typical ocular abnormalities. Paget’s disease occurs in older individuals and does not present with cafe-au-lait spots. Ollier’s disease (multiple enchondromatosis) may show bone changes but not the other findings.
REFERENCES: Albright F, Butler AM, Hampton AO, et al: Syndrome characterized by osteitis fibrosa disseminata, areas of pigmentation and endocrine dysfunction with precocious puberty in females. N Engl J Med 1937;216:727-746.
Danon M, Robboy SJ, Kim S, Scully R, Crawford JD: Cushing syndrome, sexual precocity, and polyostotic fibrous dysplasia (Albright syndrome) in infancy. J Pediatr 1975;87:917-921.
Grabias SL, Campbell CJ: Fibrous dysplasia. Orthop Clin North Am 1977;8:771-783.
81. Figure 41a shows the AP radiograph of a 15-year-old boy who reports lateral knee pain. Figures 41b and 41c show a radiograph of the distal femur that was obtained 5 years ago and a current CT scan. The indication for surgery in this patient would be
1- activity limitations because of localized pain.
2- the likelihood of development of malignant degeneration in adulthood.
3- the likelihood of development of a growth deformity.
4- ambiguity regarding the diagnosis based on the imaging studies.
5- obvious progression over the past 5 years, raising suspicion of malignant degeneration.
PREFERRED RESPONSE: 1
DISCUSSION: In a young person with solitary osteochondroma, the best surgical indication is symptoms that limit activity. A growth deformity is unlikely to occur at this age. Malignant degeneration is exceptionally rare and noted most commonly in adults. Growth is expected until skeletal maturity.
REFERENCES: Mirra JM: Bone Tumors: Clinical, Radiologic, and Pathologic Correlations. Philadelphia, PA, Lea and Febiger, 1989, pp 1626-1659.
Simon MA, Springfield DS, et al: Common Benign Bone Tumors and Usual Treatment: Surgery for Bone and Soft Tissue Tumors. Philadelphia, PA, Lippincott Raven, 1998, pp 181-205.
82. In what decade does the peak incidence of conventional osteosarcoma occur?
1- First
2- Second
3- Fourth
4- Fifth
5- Sixth
PREFERRED RESPONSE: 2
DISCUSSION: Conventional osteosarcoma most frequently occurs in the second decade, followed by the third decade. Approximately 70% to 75% of patients with osteosarcoma are between the ages of 10 and 25 years. Secondary osteosarcoma (arising in Paget’s disease or radiation-induced) is seen in older adults.
REFERENCES: Simon MA, Springfield DS, et al: Osteogenic Sarcoma: Surgery for Bone and Soft Tissue Tumors. Philadelphia, PA, Lippincott Raven, 1998, p 266.
Mirra JM: Bone Tumors: Clinical, Radiologic, and Pathologic Correlations. Philadelphia, PA, Lea and Febiger, 1989.
Wold L, et al: Osteogenic sarcoma: Atlas of Orthopaedic Pathology. Philadelphia, PA, WB Saunders, 1990, p 14.
83. A 10-year-old boy has a painful thigh mass. A radiograph, MRI scan, and biopsy specimen are shown in Figures 42a through 42c. What is the most likely diagnosis?
1- Infection
2- Parosteal osteosarcoma
3- Chondroblastic osteosarcoma
4- Chondrosarcoma
5- Osteochondroma
PREFERRED RESPONSE: 3
DISCUSSION: A destructive mixed lytic and blastic metaphyseal lesion with a large soft-tissue mass in an adolescent is most likely an osteosarcoma until proven otherwise. The epicenter of the tumor is on the surface of the bone, most likely involves the periosteum, and is more likely to be chondroblastic in nature. Parosteal osteosarcoma is a low-grade tumor, much more radiodense, usually smaller, and found in the posterior distal femur of middle-aged patients. Chondrosarcomas are distinctly rare in childhood.
REFERENCE: Bertoni F, Bacchini P: Classification of bone tumors. Eur J Radiol
1998;27:S74-S76.
84. A 21-year-old man with neurofibromatosis and multiple cutaneous neurofibromas has a rapidly enlarging painless mass on his buttock. Examination reveals a nontender, well-defined 6- x 6-cm soft-tissue mass that is deep to the fascia. The best course of action should be to order
1- MRI of the buttock.
2- CT of the buttock.
3- a needle biopsy.
4- an incisional biopsy.
5- an excisional biopsy.
PREFERRED RESPONSE: 1
DISCUSSION: Patients with neurofibromatosis are at risk for development of soft-tissue sarcomas (most commonly malignant peripheral nerve sheath tumors). Clinical indications of development of a neurofibrosarcoma include a rapidly enlarging soft-tissue mass; therefore, this patient should be considered to have a neurofibrosarcoma until proven otherwise. MRI is superior to CT in characterizing the anatomic location of soft-tissue masses and the signal characteristics of the lesion. Areas of necrosis within the tumor may be apparent on MRI that cannot be appreciated on CT, suggesting a malignant tumor. Local imaging studies of suspected malignant tumors should be performed prior to needle or open biopsy so that the biopsy site can be excised at the time of definitive resection. Additionally, postbiopsy changes may lead to MRI artifacts that alter the interpretation of the MRI.
REFERENCES: Demas BE, Heelan RT, Lane J, Marcove R, Hajdu S, Brennan MF: Soft-tissue sarcomas of the extremities: Comparison of MR and CT in determining the extent of disease. Am J Roentgenol 1988;150:615-620.
Kransdorf MJ, Jelinek JS, Moser RP Jr, et al: Soft-tissue masses: Diagnosis using MR imaging. Am J Roentgenol 1989;153:541-547.
85. A 21-year-old man has had progressive right knee pain for the past 2 months that is exacerbated with weight-bearing activities. A plain radiograph and an MRI scan are shown in Figures 43a and 43b. A biopsy specimen is shown in Figure 43c. According to the Enneking staging system of tumor classification, the lesion should be classified as what stage?
1- 1
2- 2
3- 3
4- I
5- II
PREFERRED RESPONSE: 3
DISCUSSION: The lesion is an eccentric lytic bone lesion within the epiphyseal-metaphyseal end of the proximal tibia. There is geographic destruction with a “fading border” extending to the articular cartilage. There is no matrix formation or periosteal reaction. The MRI scan shows cortical destruction with extension into the soft tissue. According to the Enneking staging system, benign lesions are stage 1, 2, or 3; malignant lesions are stage I, II, or III. Benign stage 1 lesions are latent; stage 2 are active; and stage 3 are benign aggressive. The histology shows a benign giant cell tumor. Given the cortical breakthrough shown on the MRI scan, the lesion should be classified as stage 3.
REFERENCES: Enneking WF: Clinical musculoskeletal pathology, in Enneking WF (ed): Appendix A. Gainesville, FL, Storter Publishing, 1986, pp 451-466.
Aboulafia AJ, Kennon RE, Jelinek JS: Benign bone tumors of childhood. J Am Acad Orthop Surg 1999;7:377-388.
86. What is a common clinical finding in patients with severe hypercalcemia secondary to bony metastasis?
1- Atrial fibrillation
2- Seizures
3- Urinary retention
4- Muscle weakness
5- Hyperreflexia
PREFERRED RESPONSE: 4
DISCUSSION: Increased levels of calcium are known to cause anorexia, nausea, vomiting, dehydration, muscle weakness, polyuria, and polydipsia. Treatment may include hydration, saline diuresis, and bisphosphonates.
REFERENCE: Frassica FJ, Gitelis S, Sim FH: Metastatic bone disease: General principles, pathophysiology, evaluation, and biopsy. Instr Course Lect 1992;41:293-300.
87. What cell type causes the bone destruction in metastatic lesions?
1- Osteoblast
2- Osteoclast
3- Macrophage
4- T lymphocyte
5- Null killer cells
PREFERRED RESPONSE: 2
DISCUSSION: The main consequence of tumor invading the bone is activation of both osteoblasts and osteoclasts. However, the osteoclastic effect predominates in the majority of tumors early after the invasion of bone by tumor cells, causing resorption of bone.
REFERENCES: Cramer SF, Fried L, Carter KJ: The cellular basis of metastatic bone disease in patients with lung cancer. Cancer 1981;48:2649-2660.
Clohisy DR, Palkert D, Ramnaraine ML, Pekurovsky I, Oursler MJ: Human breast cancer induces osteoclast activation and increases the number of osteoclasts at sites of tumor osteolysis. J Orthop Res 1996;14:396-402.
88. What is the most common malignant bone tumor seen in patients with multiple hereditary exostosis?
1- Dedifferentiated chondrosarcoma
2- Mesenchymal chondrosarcoma
3- Secondary chondrosarcoma
4- Clear cell chondrosarcoma
5- Periosteal osteosarcoma
PREFERRED RESPONSE: 3
DISCUSSION: Secondary chondrosarcomas are most common in patients with multiple hereditary exostosis. Dedifferentiated chondrosarcoma is less common and refers to bone lesions in which a high-grade spindle cell sarcoma component is located immediately adjacent to a low-grade cartilage neoplasm. Mesenchymal chondrosarcoma, clear cell chondrosarcoma, and periosteal osteosarcoma are no more common in patients with multiple hereditary exostosis than in the general population.
REFERENCES: Mirra JM: Bone Tumors: Clinical, Radiologic, and Pathologic Correlations. Philadelphia, PA, Lea and Febiger, 1989, pp 1660-1669.
Simon MA, Springfield DS, et al: Common Malignant Bone Tumors: Chondrosarcoma. Surgery for Bone and Soft Tissue Tumors. Philadelphia, PA, Lippincott Raven, 1998, pp 275-286.
89. An athletic 55-year-old man reports a painless mass in the anterior aspect of the thigh that appeared 3 weeks ago and has not changed in size. The patient denies any history of trauma. Examination reveals a firm, well-defined nontender mass in the anterior thigh and no inguinal adenopathy or cutaneous changes. Plain radiographs are unremarkable. T1- and T2-weighted MRI scans are shown in Figures 44a and 44b. What is the most likely diagnosis?
1- Hematoma
2- Lipoma
3- Soft-tissue sarcoma
4- Pyomyositis
5- Hemangioma
PREFERRED RESPONSE: 3
DISCUSSION: The presence of a painless soft-tissue mass that is greater than 5 cm and deep to the fascia should be considered a soft-tissue sarcoma until proven otherwise. The diagnosis of a hematoma should be made with great caution because the absence of a history of trauma, pain, or presence of ecchymosis makes it unlikely. A diagnosis of pyomyositis is unlikely because of the absence of warmth, erythema, or adenopathy. The MRI scans are not consistent with lipoma or hemangioma. The MRI signal characteristics of a lipoma should be the same as subcutaneous fat on all sequences. Soft-tissue hemangiomas are not well defined and have an infiltrative appearance on MRI scans, as does pyomyositis.
REFERENCES: Sim FH, Frassica FJ, Frassica DA: Soft-tissue tumors: Diagnosis, evaluation and management. J Am Acad Orthop Surg 1994;2:202-211.
Kransdorf MJ, Jelinek JS, Moser RP Jr, et al: Soft-tissue masses: Diagnosis using MR imaging. Am J Roentgenol 1989;153:541-547.
90. Epithelioid sarcoma most commonly occurs in which of the following anatomic locations?
1- Upper extremity
2- Lower extremity
3- Pelvis and buttock
4- Retroperitoneum
5- Spine
PREFERRED RESPONSE: 1
DISCUSSION: Epithelioid sarcoma is a rare soft-tissue sarcoma that most commonly arises in the hand or upper extremity, and it is frequently misdiagnosed as an infection or granuloma. It tends to have a higher incidence of lymph node metastasis than other soft-tissue sarcomas. The mainstay of treatment is wide surgical excision, even if amputation is necessary.
REFERENCES: Gupta TD, Chaudhuri P (eds): Tumors of the Soft Tissues, ed 2. Stamford, CT, Appleton and Lange, 1998, p 475.
Enzinger FM, Weiss SW: Soft Tissue Tumors, ed 3. St. Louis, MO, Mosby-Year Book, 1995, p 1074.
91. What common cytologic abnormality is associated with Ewing’s sarcoma?
1- t (2, 13)
2- t (x, 18)
3- t (11, 22)
4- t (12, 16)
5- t (12, 22)
PREFERRED RESPONSE: 3
DISCUSSION: Cytogenetic abnormalities have been well characterized in a number of tumors. Translocation t (2, 13), (x, 18), (12, 16), and (12, 22) have been characterized in rhabdomyosarcoma, synovial cell sarcoma, myxoid liposarcoma, and clear cell sarcoma, respectively. Translocation t(11:22) can be identified in 95% of patients with Ewing’s sarcoma. This was first described by Turc-Carel and associates in 1984.
REFERENCES: Enzinger FM, Weiss SW: Soft Tissue Tumors, ed 3. St Louis, MO, Mosby Year Book, 1995, pp 105-118.
Turc-Carel C, Philip I, Berger MP, Philip T, Lenoir GM: Chromosome study of Ewing’s sarcoma (ES) cell lines: Consistency of a reciprocal translocation t(11;22) (q24;q12). Cancer Genet Cytogenet 1984;12:1-19.
Simon MA, Springfield DS, et al: Ewing’s Sarcoma: Surgery for Bone and Soft Tissue Tumors. Philadelphia, PA, Lippincott Raven, 1998, pp 287-297.
92. Figures 45a and 45b show the radiographs of a 46-year-old man who reports the acute onset of right knee pain and is unable to bear weight on the extremity. His medical history is unremarkable. The next most appropriate step in management should consist of
1- intramedullary rod fixation.
2- arterial embolization.
3- skeletal traction and radiation therapy for 2 weeks.
4- CT of the chest, abdomen, and pelvis, and serum protein electrophoresis.
5- a skeletal survey and serum protein electrophoresis.
PREFERRED RESPONSE: 4
DISCUSSION: The patient has a pathologic fracture of the right distal femur; therefore, given the patient’s age, the most likely diagnosis is metastatic carcinoma. Staging studies should be obtained prior to surgical treatment. Immediate intramedullary fixation is contraindicated before a diagnosis is made by biopsy. Surgical stabilization should be performed prior to radiation therapy.
REFERENCE: Rougraff BT, Kneisl JS, Simon MA: Skeletal metastases of unknown origin: A prospective study of a diagnostic strategy. J Bone Joint Surg Am 1993;75:1276-1281.
93. Figure 46 shows the MRI scan of a patient who has a mass in the calf that has been fluctuating in size. Radiographs are negative. Which of the following procedures will most quickly aid in confirming the diagnosis?
1- Open biopsy
2- Needle aspiration
3- Arthroscopy
4- CT
5- Arthrography
PREFERRED RESPONSE: 2
DISCUSSION: The bright signal on the T2-weighted MRI scan suggests fluid. The multiloculated appearance in proximity to the proximal tibiofibular joint suggests that the most likely diagnosis is a ganglion. They typically increase and decrease in size and can be diagnosed by the classic gelatinous fluid obtained through needle aspiration.
REFERENCES: Bianchi S, Abdelwahab IF, Kenan S, Zwass A, Ricci G, Palomba G: Intramuscular ganglia arising from the superior tibiofibular joint: CT and MR evaluation. Skeletal Radiol 1995;24:253-256.
Feldman F, Singson RD, Staron RB: Magnetic resonance imaging of para-articular and ectopic ganglia. Skeletal Radiol 1989;18:353-358.
94. What are the five most common tumors that metastasize to bone?
1- Breast, cervical, ovarian, thyroid, lung
2- Breast, prostate, lung, renal, thyroid
3- Renal, thyroid, prostate, breast, gastrointestinal
4- Prostate, lung, breast, renal, laryngeal
5- Prostate, breast, lung, thyroid, gastrointestinal
PREFERRED RESPONSE: 2
DISCUSSION: The five most common primary carcinomas that metastasize to bone are breast, prostate, lung, renal, and thyroid in decreasing order of incidence.
REFERENCES: Frassica FJ, Gitelis S, Sim FH: Metastatic bone disease: General principles, pathophysiology, evaluation, and biopsy. Instr Course Lect 1992;41:293-300.
CA, January/February 2000, vol 50, no. 1 (Cancer Statistics).
95. Pain associated with a proximal medial tibial osteochondroma in a 10-year-old patient is most commonly the result of
1- degeneration into a chondrosarcoma.
2- bursitis.
3- infection.
4- fracture through the base.
5- associated spinal stenosis.
PREFERRED RESPONSE: 2
DISCUSSION: Pain secondary to an osteochondroma is usually from soft-tissue irritation and bursal formation. This is particularly common for proximal medial tibia osteochondromas that irritate the pes anserine tendons. Malignant degeneration into a chondrosarcoma rarely occurs, is usually associated with multiple hereditary exostoses, and usually occurs after skeletal maturity.
REFERENCES: Borges AM, Huvos AG, Smith J: Bursa formation and synovial chondrometaplasia associated with osteochondromas. Am J Clin Pathol 1981;75:648-653.
Hudson TM, Springfield DS, Spanier SS, Enneking WF, Hamlin DJ: Benign exostoses and exostotic chondrosarcomas: Evaluation of cartilage thickness by CT. Radiology 1984;152:595-599.
96. Figures 47a through 47f show the AP radiograph, bone scan, CT scan, MRI scan, and biopsy specimens of a 30-year-old woman who has had vague left shoulder pain for
1 year. Management should consist of
1- curettage.
2- radiation therapy.
3- radiation therapy and chemotherapy.
4- antibiotics.
5- wide resection.
PREFERRED RESPONSE: 1
DISCUSSION: The histology shows eosinophils with a background of larger cells (Langerhans’ cells). This is consistent with eosinophilic granuloma. Localized sites are best treated with curettage, steroid injection, or observation. Chemotherapy is used only if there is systemic involvement.
REFERENCES: Mirra JM: Eosinophilic granuloma, in Bone Tumors: Clinical, Radiologic, and Pathologic Correlations. London, England, Lea and Febiger, 1989, pp 1023-1060.
Sessa S, Sommelet D, Lascombes P, Prevot J: Treatment of Langerhans-cell histiocytosis in children: Experience at the Children’s Hospital of Nancy. J Bone Joint Surg Am 1994;76:1513-1525.
Mickelson MR, Bonfiglio M: Eosinophilic granuloma and its variations. Orthop Clin North Am 1977;8:933-945.
97. What is the 5-year overall survival rate for adults with high-grade soft-tissue sarcomas?
1- 0%
2- 30%
3- 50%
4- 70%
5- 90%
PREFERRED RESPONSE: 3
DISCUSSION: The 5-year overall survival rate for deep, high-grade soft-tissue sarcomas is around 50%. The overall survival and disease-free survival rates chiefly depend on the tumor stage, but for all stages combined, most cancer treatment centers report a 5-year overall survival rate of around 70% and a disease-free survival rate of 65%.
REFERENCES: Fleming ID, et al: Manual for Staging of Cancer/American Joint Committee on Cancer, ed 5. Philadelphia, PA, Lippincott Raven, 1997, pp 149-156.
Cheng EY, Dusenbery KE, Winters MR, Thompson RC: Soft tissue sarcomas: Preoperative versus postoperative radiation therapy. J Surg Oncol 1996;61:90-99.
98. Figures 48a through 48c show the lateral radiograph and MRI scans of a 60-year-old man who has had pain in his thigh for 1 month. The next most appropriate step in management should consist of
1- insertion of a locked intramedullary nail.
2- a bone scan, a radiograph of the chest, and CT of the chest, abdomen, and pelvis.
3- compression plating.
4- upper and lower gastrointestinal (GI) studies.
5- radiation therapy.
PREFERRED RESPONSE: 2
DISCUSSION: The patient has a presumed metastatic bone tumor. The approach to evaluating a patient with a bone tumor of unknown primary origin is to obtain laboratory studies that include a CBC, an erythrocyte sedimentation rate, a serum protein electrophoresis, a calcium level, a urinalysis, and a prostate-specific antigen. In addition, a bone scan, a radiograph of the chest, and CT scans of the chest and abdomen should be obtained. These evaluations can identify the primary site in 85% of patients. GI studies rarely are of diagnostic value. Prophylactic stabilization is contraindicated until a diagnosis is confirmed by histology.
REFERENCES: Frassica FJ, Frassica DA, McCarthy EF, Riley LH III: Metastatic bone disease: Evaluation, clinicopathologic features, biopsy, fracture risk, nonsurgical treatment, and supportive management. Instr Course Lect 2000;49:453-459.
Rougraff BT, Kneisl JS, Simon MA: Skeletal metastases of unknown origin: A prospective study of a diagnostic strategy. J Bone Joint Surg Am 1993;75:1276-1281.
99. A 17-year-old boy has had a mass in his right thigh for the past 6 months. He denies any history of trauma. Examination reveals that the mass is painless and firm. A radiograph and axial MRI scan are shown in Figures 49a and 49b. What is the most likely diagnosis?
1- Osteochondroma
2- Osteosarcoma
3- Periosteal chondroma
4- Ewing’s sarcoma of bone
5- Chondroblastoma
PREFERRED RESPONSE: 1
DISCUSSION: Osteochondroma typically occurs as a bony projection or a sessile growth on the bone, and there can be flaring of the metaphysis. The radiograph shows continuity from the adjacent cancellous bone into the lesion itself. There is no soft-tissue mass or bone destruction to suggest osteosarcoma or Ewing’s sarcoma. Periosteal chondroma has a scalloped out radiographic appearance. Chondroblastoma typically is an epiphyseal-based lesion.
REFERENCE: Wold LA, et al: Osteochondroma: Atlas of Orthopaedic Pathology. Philadelphia, PA, WB Saunders, 1990, p 50.
100. Evaluation of the percent of necrosis in the resected specimen after preoperative chemotherapy is of prognostic value for what type of sarcoma?
1- Chondrosarcoma
2- Soft-tissue sarcoma
3- Rhabdomyosarcoma
4- Parosteal osteosarcoma
5- Osteosarcoma
PREFERRED RESPONSE: 5
DISCUSSION: To date, only the percent of necrosis after induction chemotherapy in high-grade osteosarcomas seems to be of prognostic value. The value in soft-tissue sarcoma and rhabdomyosarcoma is being evaluated but has not been substantiated. Chondrosarcomas and parosteal osteosarcomas are not treated with chemotherapy.
REFERENCES: Rosen G, Marcove RC, Caparros B, Nirenberg A, Kosloff C, Huvos AG: Primary osteogenic sarcoma: The rationale for pre-operative chemotherapy and delayed surgery. Cancer 1979,43:2163-2177.
Davis AM, Bell RS, Goodwin PJ: Prognostic factors in osteosarcoma: A critical review. J Clin Oncol 1994;12:423-431.
Wunder JS, Paulian G, Huvos AG, Heller G, Meyers PA, Healey JH: The histological response to chemotherapy as a predictor of the oncological outcome of operative treatment of Ewing sarcoma. J Bone Joint Surg Am 1998;80:1020-1033.
FOR ALL MCQS CLICK THE LINK ORTHO MCQ BANK